13.02.2014 Aufrufe

Spektren, Auswahlregeln und das Wigner-Eckart-Theorem

Spektren, Auswahlregeln und das Wigner-Eckart-Theorem

Spektren, Auswahlregeln und das Wigner-Eckart-Theorem

MEHR ANZEIGEN
WENIGER ANZEIGEN

Sie wollen auch ein ePaper? Erhöhen Sie die Reichweite Ihrer Titel.

YUMPU macht aus Druck-PDFs automatisch weboptimierte ePaper, die Google liebt.

Universität Stuttgart<br />

Institut für Theoretische <strong>und</strong> Angwandte Physik<br />

<strong>Spektren</strong>, <strong>Auswahlregeln</strong> <strong>und</strong><br />

<strong>das</strong> <strong>Wigner</strong>-<strong>Eckart</strong>-<strong>Theorem</strong><br />

Hauptseminar: Gruppen in der Physik<br />

Boris Lander<br />

9. Januar 2008


Inhaltsverzeichnis<br />

1 Einleitung 2<br />

2 Gr<strong>und</strong>lagen der Quantenmechanik 3<br />

2.1 Was führte überhaupt zur Entwicklung der Quantenmechanik? 3<br />

2.2 Quantenmechanische Postulate . . . . . . . . . . . . . . . . . 3<br />

2.3 Schrödingergleichung . . . . . . . . . . . . . . . . . . . . . . . 5<br />

2.4 Eigenwertspektrum & Quantenzahlen . . . . . . . . . . . . . . 6<br />

2.5 Übergänge zwischen den Niveaus . . . . . . . . . . . . . . . . 7<br />

2.5.1 Dipol-Übergänge . . . . . . . . . . . . . . . . . . . . . 8<br />

2.5.2 Raman-Übergänge . . . . . . . . . . . . . . . . . . . . 9<br />

2.6 Störungsrechnung . . . . . . . . . . . . . . . . . . . . . . . . . 11<br />

2.6.1 Zeitunabhängige Störungsrechung . . . . . . . . . . . . 12<br />

2.6.2 Zeitabhängige Störungsrechnung . . . . . . . . . . . . 14<br />

3 Produkträume 17<br />

4 Tensoroperatoren 20<br />

4.1 Denition . . . . . . . . . . . . . . . . . . . . . . . . . . . . . 20<br />

4.2 Beispiel: Ortsoperator . . . . . . . . . . . . . . . . . . . . . . 21<br />

5 <strong>Wigner</strong>-<strong>Eckart</strong>-<strong>Theorem</strong> 24<br />

6 Physikalische Anwendungen des <strong>Wigner</strong>-<strong>Eckart</strong>-<strong>Theorem</strong>s 27<br />

6.1 <strong>Auswahlregeln</strong> . . . . . . . . . . . . . . . . . . . . . . . . . . . 27<br />

6.1.1 Elektromagnetische Dipolübergänge . . . . . . . . . . 27<br />

6.1.2 Ramanübergänge . . . . . . . . . . . . . . . . . . . . . 35<br />

6.2 Eigenwertprobleme . . . . . . . . . . . . . . . . . . . . . . . . 37<br />

7 Zusammenfassung 40<br />

1


1 Einleitung<br />

Eine, besonders in der Quantenphysik, sehr häug auftretende Problemstellung<br />

ist die Berechnung von sog. Matrixübergangselementen, deren Betragsquadrate<br />

im Wesentlichen die Wahrscheinlichkeit dafür beschreiben mit der<br />

beispielsweise ein optischer Übergang eines Atoms stattndet. Ohne weitere<br />

Vereinfachung müssten hier sehr viele Matrixelemente berechnet werden.<br />

Da diese Probleme jedoch sehr oft eine bestimmte Symmetrie aufweisen,<br />

kann man unter deren Ausnutzung erhebliche Vereinfachungen erzielen. Und<br />

genau hier setzt <strong>das</strong> <strong>Wigner</strong>-<strong>Eckart</strong>-<strong>Theorem</strong> an. Mit dessen Hilfe können<br />

diese Symmetrien ausgenutzt werden um vorab schon Aussagen über <strong>das</strong><br />

Verschwinden, oder auch über die Verhältnisse der Intensitäten zwischen<br />

verschiedenen Matrixübergangselementen zu machen. Aus dem Verschwinden<br />

solcher Matrixelemente können dann <strong>Auswahlregeln</strong> abgeleitet werden,<br />

die besonders für die Spektroskopie von groÿer Bedeutung sind.<br />

2


2 Gr<strong>und</strong>lagen der Quantenmechanik<br />

Um sich mit diesen Problemstellungen genauer auseinander setzen zu können<br />

ist es zunächst nötig sich, zumindest in Gr<strong>und</strong>zügen, mit den Ergebnissen<br />

der Quantenmechanik vertraut zu machen.<br />

2.1 Was führte überhaupt zur Entwicklung der Quantenmechanik?<br />

Im Rahmen der klassischen Physik lassen sich einige Ergebnisse physikalischer<br />

Experimente nicht erklären. Beispiele hierfür sind:<br />

• Die Diskretheit atomarer Spektrallinien<br />

• Die Frequenzverteilung der Hohlraumstrahlung<br />

• Die Wellennatur materieller Teilchen<br />

All diese Probleme werden mit Hilfe der Quantenmechanik widerspruchsfrei<br />

erklärbar.<br />

Da die Ergebnisse der klassischen Mechanik in vielen Bereichen der Natur<br />

jedoch durch Experimente sehr gut bestätigt wurden, müssen auch die Ergebnisse<br />

der Quantenphysik im klassischen Grenzfall ( im "Grenzfall groÿer<br />

Quantenzahlen"bzw. → 0), diese bereits verizierten Ergebnisse liefern.<br />

Man bezeichnet dies als "Korrespondenzprinzip".<br />

2.2 Quantenmechanische Postulate<br />

Die gr<strong>und</strong>legenden Werkzeuge, die zum Verständnis der Quantenmechanik<br />

nötig sind, sind die quantenmechanischen Postulate. Diese sollen Gegenstand<br />

dieses Abschnitts sein.<br />

Postulat 1 (Quantenmechanische Zustände) Der Quantenmechanische<br />

Zustand eines Teilchens wird durch eine komplexwertige Wellenfunktion<br />

Ψ(r, t) beschrieben. Diese sind Elemente des Hilbertraums der quadratintegrablen<br />

Funktionen V über C, auf dem ein komplexes Skalarprodukt deniert<br />

ist.<br />

3


Postulat 2 (Superpositionsprinzip) Jede Linearkombination von quantenmechanischen<br />

Zuständen ist wieder ein möglicher quantenmechanischer<br />

Zustand. Es gilt also <strong>das</strong> Superpositionsprinzip.<br />

Postulat 3 (Kopenhagener Interpretation) Das Betragsquadrat einer<br />

Wellenfunktion |Ψ(r, t)| 2 ist als Wahrscheinlichkeitsdichte zu interpretieren.<br />

D.h. |Ψ(r, t)| 2 dV ist die Wahrscheinlichkeit dafür, <strong>das</strong> Teilchen zur Zeit t in<br />

einem Volumenelement dV um r anzutreen.<br />

Postulat 4 (Physikalische Observablen) Physikalische Observablen, also<br />

experimentell beobachtbare Gröÿen, werden in der Quantenmechanik durch<br />

hermitesche Operatoren dargestellt. Die Messwerte sind die Eigenwerte des<br />

zugehörigen Operators. (D.h. wenn sich ein quantenmechanisches System<br />

nicht in einem Eigenzustand bendet, sondern in einer Linearkombination<br />

aus solchen, so wird man es bei Messung einer Observablen immer in genau<br />

einem dieser Eigenzustände vornden. Man beeinusst also durch die<br />

Messung <strong>das</strong> System, an dem man diese durchführt, selbst.)<br />

Postulat 5 (Schrödingergleichung) Die zeitliche Entwicklung eines<br />

quantenmechanischen Zustands wird durch die zeitabhängige Schrödingergleichung<br />

beschrieben.<br />

ĤΨ(x, t) = i ∂ Ψ(x, t)<br />

∂t<br />

Postulat 6 (Messwerte) Sei Ô ein hermitescher Operator mit den Eigenfunktionen<br />

ϕ n (r) zu den Eigenwerten o n . Und sei ϕ(r) = ∑ n c nϕ n (r) der<br />

Zustand eines quantenmechanischen Systems. Dann ist |c n | 2 die Wahrscheinlichkeit<br />

dafür, bei einer Messung der Observablen Ô an diesem System, den<br />

Wert o n zu messen. D.h. <strong>das</strong> System ist mit einer Wahrscheinlichkeit von<br />

|c n | 2 im Zustand ϕ n (r) anzutreen.<br />

4


2.3 Schrödingergleichung<br />

Die zentrale Gleichung, mit der ein quantenmechanisches System beschrieben<br />

werden kann, ist die Schrödingergleichung. Diese lässt sich nicht streng<br />

mathematisch herleiten <strong>und</strong> ist daher als Postulat zu verstehen (siehe oben).<br />

Sie lautet:<br />

ĤΨ(x, t) = i ∂ Ψ(x, t) (1)<br />

∂t<br />

In der Ortsdarstellung (ˆx = x, ˆp = −i∇) hat der Hamiltonoperator die<br />

Form:<br />

Ĥ = − 2<br />

∆ + V (r, t) (2)<br />

2m<br />

Damit lautet die Schrödingergleichung in der Ortsdarstellung:<br />

)<br />

(− 2<br />

2m ∆ + V (r, t) Ψ(r, t) = i ∂ Ψ(r, t) (3)<br />

∂t<br />

Hier ist Ψ(x, t) die Wellenfunktion, Ĥ der Hamiltonoperator <strong>und</strong> =<br />

h<br />

2π<br />

mit dem Planckschen Wirkungsquantum h. Die Schrödingergleichung ist eine<br />

lineare Dierentialgleichung (→ Superpositionsprinzip) zweiter Ordnung in<br />

den Ortsvariablen <strong>und</strong> erster Ordnung in der Zeit.<br />

Nimmt man nun an, <strong>das</strong>s der Hamiltonoperator zeitunabhängig ist, (wenn<br />

also V (r, t) = V (r),) so ist für die Wellenfunktion ein Separationsansatz<br />

folgender Form möglich:<br />

Ψ(r, t) = ϕ(r)f(t) (4)<br />

Eingesetzt in die zeitabhängige Schrödingergleichung ergibt sich:<br />

Umgeformt erhält man:<br />

] [Ĥϕ(r) f(t) = ϕ(r)i ∂ f(t) (5)<br />

∂t<br />

Ĥϕ(r)<br />

ϕ(r)<br />

= i ∂ ∂t f(t)<br />

f(t)<br />

= Konstante = E (6)<br />

5


Da die linke Seite der Gleichung nun nur noch vom Ort <strong>und</strong> die rechte nur<br />

noch von der Zeit abhängt, die Gleichung aber für alle unabhängig voneinander<br />

gewählten r <strong>und</strong> t gelten muss, muss jede Seite der Gleichung für sich<br />

konstant sein. Für die Zeitfunktion gilt dann:<br />

f(t) = exp (− i Et) (7)<br />

<br />

Der ortsabhängige Teil der Wellenfunktion muss dann die zeitunabhängige<br />

Schrödingergleichung erfüllen:<br />

Ĥϕ(r) = Eϕ(r) (8)<br />

Dies ist eine Eigenwertgleichung in der ϕ(r) die Eigenfunktionen <strong>und</strong> E die<br />

Eigenwerte darstellen. E ist also der Eigenwert zum Hamiltonoperator. Dieser<br />

beschreibt die Energie des quantenmechanischen Zustandes. Der Hamiltonoperator<br />

ist der, der Observable "Energie"zugeordnete, Operator.<br />

Wie die Wellenfunktion physikalisch zu interpretieren ist, wurde bereits in<br />

Postulat 3 beschrieben. Die Wellenfunktion ist im Allgemeinen eine komplexwertige<br />

Funktion. Da physikalisch messbare Gröÿen aber reell sein müssen,<br />

hat die Wellenfunktion selbst also keine physikalische Bedeutung. Ihr Betragsquadrat<br />

jedoch sehrwohl. Dieses ist nach Postulat 3 physikalisch als<br />

Aufenthaltswahrscheinlichkeitsdichte des beschriebenen Teilchens zu verstehen.<br />

Daraus folgt sofort die Normierungsbedingung:<br />

∫<br />

V<br />

Ψ ∗ ΨdV = 1 (9)<br />

Denn die Wahrscheinlichkeit dafür, <strong>das</strong> Teilchen irgendwo im gesamten relevanten<br />

Volumen zu nden ist 1, da <strong>das</strong> Teilchen ja irgendwo sein muss.<br />

Diese Normierungsbedingung muss als Randbedingung zusätzlich zur Schrödingergleichung<br />

erfüllt sein.<br />

2.4 Eigenwertspektrum & Quantenzahlen<br />

Die Schrödingergleichung ist für verschiedene einfache Potentiale exakt lösbar.<br />

Beispiele hierfür sind:<br />

• Harmonisches Potential<br />

• Kastenpotential<br />

• Potentialbarriere<br />

6


• Radialsymmetrisches Coulombpotential<br />

Für geb<strong>und</strong>ene/ungeb<strong>und</strong>ene Zustände ergibt sich in der Regel ein diskretes/kontinuierliches<br />

Eigenwertspektrum. D.h. im geb<strong>und</strong>enen Fall ist die Lösung<br />

der Schrödingergleichung nur für bestimmte, diskrete Energien möglich.<br />

Jedem dieser Energiewerte kann man dann eine Zahl (→ Quantenzahl) zuordnen.<br />

In der Quantenmechanik gilt folgender Satz:<br />

Satz 1 (Kommutierende Operatoren) Zwei Observablen besitzen genau<br />

dann einen vollständigen, gemeinsamen Satz orthonormaler Eigenfunktionen<br />

(simultane Eigenfunktionen), wenn sie kommutieren.<br />

Gibt es also Observablen, deren Operatoren miteinander kommutieren, so<br />

kann man ein vollständiges, simultanes System von Eigenfunktionen wählen.<br />

So gewählt sind diese Funktionen Eigenfunktion zu jeder dieser Observablen.<br />

Wenn sich also ein System in einem solchen Eigenzustand bendet, kann man<br />

all diese Observabeln messen, ohne <strong>das</strong> System zu beeinussen. Observablen<br />

deren Operatoren vertauschen lassen sich also gleichzeitig beobachten.<br />

Auÿerdem kann man dem System Quantenzahlen bezüglich jeder dieser Observablen<br />

zuordnen. Man nennt diese Quantenzahlen "gute Quantenzahlen".<br />

Wird <strong>das</strong> System so gestört, <strong>das</strong>s die Operatoren nicht mehr vertauschen,<br />

so gibt es keine simultanen Eigenfunktionen mehr <strong>und</strong> die Quantenzahlen<br />

werden zu "schlechten Quantenzahlen".<br />

Gibt man alle guten Quantenzahlen an, so ist die Wellenfunktion (bis auf<br />

Phasenfaktoren) eindeutig bestimmt. Das heiÿt also, <strong>das</strong>s sich jeder quantenmechanische<br />

Zustand durch die Angabe aller guten Quantenzahlen charakterisieren<br />

lässt.<br />

2.5 Übergänge zwischen den Niveaus<br />

In einem quantenmechanischen System können nun Übergänge zwischen<br />

den verschiedenen diskreten Energieniveaus stattnden. Dabei wird entweder<br />

Energie aufgenommen, dann geht <strong>das</strong> System in einen Zustand höherer<br />

Energie über, oder es wird Energie abgegeben, dann geht <strong>das</strong> System in einen<br />

Zustand niedrigerer Energie über. Im Folgenden sollen hier zwei spezielle Arten<br />

dieser Übergänge betrachtet werden.<br />

7


2.5.1 Dipol-Übergänge<br />

Bei Dipol-Übergängen handelt es sich um Übergänge, bei denen Photonen<br />

absorbiert oder emittiert werden. Die Energie dieser Photonen (E = ω)<br />

entspricht dabei genau der Energiedierenz der quantenmechanischen Zustände.<br />

Beobachtet man solche Übergänge bei Atomen, so liegen diese Energien<br />

gröÿenordnungsmäÿig häug im Bereich von wenigen eV, also im optisch<br />

sichtbaren Bereich.<br />

Man unterscheidet bei solchen Übergängen zwischen drei Fällen:<br />

• Absorption: Bei der Absorption trit ein Photon der Energie ω =<br />

E 2 − E 1 auf <strong>das</strong> System, wird von diesem absorbiert <strong>und</strong> regt dabei<br />

<strong>das</strong> System vom Zustand 1 mit der Energie E 1 in den Zustand 2 mit<br />

der Energie E 2 an.<br />

• Induzierte Emission: Die induzierte Emission entspricht gerade dem<br />

inversen Prozess der Absorption. Auch hier trit ein Photon der Energie<br />

ω = E 2 − E 1 auf <strong>das</strong> System, <strong>das</strong> sich jetzt aber in Zustand 2 be-<br />

ndet. Das Photon bewirkt nun den Übergang des Systems in Zustand<br />

1 unter Emission eines Photons, <strong>das</strong> in Energie <strong>und</strong> Phase identisch zu<br />

dem Photon ist, <strong>das</strong> den Übergang ausgelöst hat.<br />

• Spontane Emission: Die spontane Emission unterscheidet sich von<br />

den beiden anderen Fällen. Nach einer gewissen mittleren Zeit geht ein<br />

System, <strong>das</strong> sich in einem angeregten Zustand bendet, ohne äuÿeren<br />

Einuss, in einen energetisch tiefer gelegenen Zustand über <strong>und</strong> sendet<br />

dabei ein Photon der Energie ω = E 2 − E 1 aus, dessen Phase jedoch<br />

zufällig ist.<br />

8


Solche Übergänge sind nicht zwischen beliebigen Quantenzuständen möglich,<br />

sondern es gibt sogenannte <strong>Auswahlregeln</strong>, die verschiedene Übergänge<br />

verbieten. Auÿerdem nden nicht alle Übergänge mit der gleichen Wahrscheinlichkeit<br />

statt. Wie man im Folgenden sehen wird, kann man mit Hilfe<br />

des <strong>Wigner</strong>-<strong>Eckart</strong>-<strong>Theorem</strong>s Aussagen über die Verhältnisse <strong>und</strong> <strong>das</strong> Verschwinden<br />

von Übergangswahrscheinlichkeiten treen.<br />

2.5.2 Raman-Übergänge<br />

Eine wichtige Methode zur Untersuchung von Molekülen ist die Raman-<br />

Spektroskopie. Der Raman-Eekt beruht auf der inelastischen Streuung von<br />

Licht an Molekülen. Inelastische Streuung bedeutet, <strong>das</strong>s bei der Streuung<br />

Energie übertragen wird. Das also die Energie des gestreuten Photons verschieden<br />

von der Energie des einfallenden Photons ist. Dies macht sich in<br />

der spektralen Verteilung des gestreuten Lichtes durch Linien bemerkbar,<br />

deren Energie sich von der, der elastisch gestreuten Rayleigh-Linie unterscheidet.<br />

Diese Energieunterschiede hängen allein vom streuenden Molekül<br />

<strong>und</strong> nicht von der Wellenlänge des einfallenden Lichts ab. Die Messung dieser<br />

Energiedierenzen erlaubt es dann Rückschlüsse auf die Molekülstruktur zu<br />

ziehen, denn diese Energiedierenzen entsprechen genau den Schwingungs<strong>und</strong><br />

Rotationsenergien der streuenden Moleküle.<br />

In der klassischen Optik erklärt man die elastische Streuung von Licht (Rayleigh-Streuung)<br />

dadurch, <strong>das</strong>s <strong>das</strong> elektrische Feld E des Lichtes an den<br />

Hüllenelektronen des Moleküls angreift <strong>und</strong> dort ein Dipolmoment induziert<br />

p ind = αE. Dabei ist α die Polarisierbarkeit des Moleküls. Diese ist im<br />

Allgemeinen anisotrop <strong>und</strong> kann muss daher durch einen Tensor zweiter Stufe<br />

beschrieben werden. Dieses Dipolmoment schwingt dann mit der Frequenz<br />

des Lichtes <strong>und</strong> emittiert deshalb selbst Dipolstrahlung mit der gleichen<br />

Frequenz wie <strong>das</strong> anregende Licht.<br />

Bei der Ramanstreuung unterscheidet man zwischen der Schwingungs- <strong>und</strong><br />

der Rotations-Raman-Streuung. Zuerst soll hier eine sehr anschauliche klassische<br />

Erklärung für den Raman-Eekt gegeben werden, die einige (aber<br />

nicht alle) Beobachtungen gut erklärt. Danach soll die quantenmechanische<br />

Erklärung noch angerissen werden.<br />

Bei der Schwingungs-Raman-Streuung wird zunächst angenommen, <strong>das</strong>s<br />

<strong>das</strong> Molekül nicht rotiert, sondern ausschlieÿlich schwingt. Bei der klassischen<br />

Erklärung geht man davon aus, <strong>das</strong>s <strong>das</strong> eingestrahlte Primärlicht der<br />

Frequenz ω p im Molekül ein Dipolmoment p ind = αE induziert. Da jedoch<br />

die Polarisierbarkeit in der Regel von den Bindungslängen abhängt, wird<br />

dieses durch eine Schwingung mit der Schwingungsfrequenz ω vib periodisch<br />

verändert. Durch die zeitlich veränderliche Polarisierbarkeit wird nun <strong>das</strong><br />

Dipolmoment, welches mit der Frequenz des Primärlichtes schwingt, durch<br />

9


die Schwingungsfrequenz des Moleküls moduliert. Für <strong>das</strong> Beispiel eines linearen<br />

zweiatomigen Moleküls mit Bindungslänge R gilt in erster Ordnung<br />

für die Polarisierbarkeit längs der Schwingungsachse:<br />

α(R) = α(R 0 ) + dα<br />

dR (R − R 0) + . . . (10)<br />

Damit erhält man für <strong>das</strong> induzierte Dipolmoment:<br />

(<br />

p ind (t) = α(t)E 0 cos(ω p t) = α(R 0 ) + dα )<br />

dR (R − R 0) E 0 cos(ω p t) (11)<br />

Für die Molekülschwingung gilt:<br />

Eingesetzt <strong>und</strong> umgeformt erhält man:<br />

p(t) = α(R 0 )E 0 cos(ωt) + 1 dα<br />

2<br />

R − R 0 = a cos(ω vib t) (12)<br />

dR E 0a (cos((ω p + ω vib )t) + cos((ω p − ω vib )t))<br />

(13)<br />

Man erkennt also neben dem Term, der mit der Primärfrequenz schwingt,<br />

zwei weitere Terme, die mit den um ω vib verschobenen Frequenzen ω p ± ω vib<br />

schwingen. Dabei bezeichet man die zu niedrigeren Frequenzen vorschobene<br />

Linie als Stokes-Linie, die zu höheren Frequenzen verschobene als Anti-<br />

Stokes-Linie. Berücksichtigt man in der Reihenentwicklung noch weitere Terme,<br />

so ergeben sich noch weitere Schwingungen der Frequenz ω p ± 2ω vib ,<br />

ω p ± 3ω vib , . . . Die Intensitäten nehmen jedoch mit zunehmender Ordnung<br />

ab. Der Schwingungs-Raman-Eekt tritt nicht auf, wenn die Polarisierbarkeit<br />

nicht von den Bindungslängen abhängt. Für den Fall des zweiatomigen<br />

Moleküls bedeutet dies in erster Ordnung dα<br />

dR = 0.<br />

Zwar liefert die klassische Theorie eine sehr anschauliche Erklärung des Effekts,<br />

wenn man sich jedoch für die Intensitäten der verschiedenen Linien<br />

interessiert, liefert die Theorie falsche Ergebnisse. Denn nach der klassischen<br />

Theorie müssten die Stokes- <strong>und</strong> die Anti-Stokes-Linie die gleiche Intensität<br />

haben. Dies ist jedoch, besonders für niedrige Temperaturen, nicht der Fall.<br />

Die Anti-Stokes-Linie hat eine höhere Energie als die Unverschobene, daher<br />

muss die Energie vom Moleküle abgegeben werden können. Dies ist nur der<br />

Fall, wenn sich <strong>das</strong> Molekül bereits in einem angeregten Zustand bendet.<br />

Da jedoch im thermischen Gleichgewicht die Wahrscheinlichkeit dafür, <strong>das</strong>s<br />

sich <strong>das</strong> Molekül in einem angeregten Zustand bendet immer kleiner ist als<br />

die, <strong>das</strong>s es sich im Gr<strong>und</strong>zustand bendet, muss die Intensität der Stokes-<br />

Linie gröÿer sein als die der Anti-Stokes-Linie. Für groÿe Temperaturen wird<br />

der Unterschied jedoch immer kleiner.<br />

Bei der Rotations-Raman-Streuung beobachtet man zur Primärfrequenz<br />

verschobene Linien, deren Energieunterschied zur Primärlinie hier jedoch<br />

10


Rotationsquanten entsprechen. Auch hier liefert ein klassisches Modell eine<br />

anschauliche Erklärung, mit der sich der Eekt zumindest in den Gr<strong>und</strong>zügen<br />

verstehen lässt. Da die Polarisierbarkeit eines nicht-rotationssymmetrischen<br />

Moleküls anisotrop ist, muss man diese als Tensor zweiter Stufe behandeln.<br />

Nun sollen die beiden Haupt-Polarisierbarkeiten mit α ⊥ <strong>und</strong> α ‖ bezeichnet<br />

werden. Dabei stehen α ‖ <strong>und</strong> α ⊥ für die lange <strong>und</strong> die kurze Achse des symmetrischen<br />

Polarisierbarkeitstensors. Meistens weist dabei α ‖ in Richtung<br />

der Figurenachse des Moleküls <strong>und</strong> α ⊥ in eine Richtung senkrecht dazu.<br />

Wenn nun <strong>das</strong> Molekül rotiert, verändert sich also die Polarisierbarkeit mit<br />

der doppelten Rotationsfrequenz des Moleküls. → Die doppelte Frequenz, da<br />

die Polarisierbarkeit schon nach einer halben Umdrehung wieder die gleiche<br />

Polarisierbarkeit vorliegt wie am Anfang. Auch hier wird durch die veränderliche<br />

Polarisierbarkeit <strong>das</strong> induzierte Dipolmoment noch zusätzlich moduliert.<br />

Analog zum Schwingungs-Raman-Eekt treten im Frequenzspektrum<br />

zusätzlich zur unverschobenen Linie noch um die doppelte Rotationsfrequenz<br />

zu höheren <strong>und</strong> zu tieferen Frequenzen verschobene Linien auf.<br />

Der Rotations-Raman-Eekt tritt nur auf, wenn α ⊥ ≠ α ‖ . Da die Rotationsenergien<br />

niedriger liegen als die Schwingungsenergien, sind die Verschiebungen<br />

kleiner als die beim Schwingungs-Raman-Eekt. Aus dem selben<br />

Gr<strong>und</strong> sind bei Normaltemperatur auch höhere Zustände angeregt, daher ist<br />

der Intensitätsunterschied zwischen den Stokes- <strong>und</strong> den Anti-Stokes-Linien<br />

wesentlich kleiner als beim Schwingungs-Raman-Eekt.<br />

Ähnlich den Dipolübergängen sind auch bei den Ramanübergängen nicht alle<br />

Übergänge erlaubt. Es gibt also auch hier <strong>Auswahlregeln</strong>, die sich im Fehlen<br />

bestimmter Linien im Spektrum ausdrücken.<br />

Die quantenmechanische Erklärung soll hier nur angerissen werden. Hier<br />

stellt man sich den Vorgang folgendermaÿen vor (vgl. Abbildung 1): Ein<br />

quantemechanisches System bendet sich im Zustand E ′ , durch Absorption<br />

eines Photons der Energie E wird es in ein sehr kurzlebiges, virtuelles<br />

Energieniveau E virtuell angehoben. (Für sehr kurze Zeit ist ein solches virtuelles<br />

Energieniveau gemäÿ der Energie-Zeit-Unschärferelation denkbar.) Von<br />

dort geht <strong>das</strong> System unter Emission eines Photons in einen Zustand der<br />

Energie E ′′ über. Das ausgesandte Photon hat die Energie E − (E ′′ − E ′ ).<br />

Dieser Prozess wird Stokes-Prozess genannt. Den umgekehrten Prozess E ′′ →<br />

E virtuell → E ′ nennt man Anti-Stokes-Prozess. Durch die von der Temperatur<br />

abhängigen Besetzungszahlen lassen sich nun auch die unterschiedlichen<br />

Intensitäten von Stokes- <strong>und</strong> Anti-Stokes-Linien erklären.<br />

2.6 Störungsrechnung<br />

Da die Schrödingergleichung nur in wenigen einfachen Fällen exakt lösbar<br />

ist, bedient man sich verschiedener Näherungsverfahren. Eines davon ist die<br />

11


Abbildung 1: Ramaneekt<br />

Störungsrechnung. Sie ist nur dann anwendbar, wenn man sich den Hamiltonoperator<br />

eines Systems als Summe eines Hamiltonoperators, für den die<br />

Lösung der Schrödingergleichung bereits bekannt ist <strong>und</strong> einem Störoperator,<br />

dessen Auswirkungen im Vergleich zum anderen Summanden nur sehr<br />

klein sind, denken kann. Wie <strong>das</strong> genau zu verstehen ist wird im Folgenden<br />

geklärt. Man unterscheidet zwei wichtige Fälle: Die zeitunabhängige <strong>und</strong> die<br />

zeitabhängige Störungsrechnung.<br />

2.6.1 Zeitunabhängige Störungsrechung<br />

1. Nicht-entarteter Ausgangszustand:<br />

Wir wollen nun ein System mit dem Hamilton-Operator<br />

H(λ) = H 0 + Ṽ (14)<br />

betrachten. Dabei nehmen wir an, <strong>das</strong>s die Eigenfunktionen <strong>und</strong> Eigenwerte<br />

zum Operator H 0 bekannt sind. Weiter nehmen wir an, <strong>das</strong>s<br />

sich Ṽ durch λV ausdrücken lässt, wobei λ eine dimensionslose sehr<br />

kleine reelle Zahl ist <strong>und</strong> die charakteristischen Energien von V <strong>und</strong><br />

H 0 vergleichbar sind. Es soll also Folgendes gelten:<br />

H = H 0 + λV mit λ ≪ 1 (15)<br />

Weiterhin soll angenommen werden, <strong>das</strong>s <strong>das</strong> Eigenwertspektrum von<br />

H 0 diskret ist <strong>und</strong> die Eigenzustände einen vollständigen orthonormierten<br />

Satz von Eigenfunktionen bilden. Also:<br />

∣<br />

H 0 ϕ i ∣<br />

p〉<br />

= E<br />

0<br />

p ϕ i 〉<br />

p<br />

(16)<br />

sowie<br />

<strong>und</strong><br />

∑ ∣<br />

∣ϕ i 〈 ∣<br />

p〉<br />

ϕ<br />

i<br />

p = id (17)<br />

p,i<br />

〈 ∣<br />

ϕ<br />

i<br />

p ϕ j 〉<br />

q = δi,j δ p,q (18)<br />

12


Dabei gibt p an, zu welchem Eigenwert ∣ ∣ ϕ<br />

i<br />

p<br />

〉 gehört <strong>und</strong> i ist der Entartungsindex,<br />

mit i = 1, . . . , g p , wenn g p der Entartungsgrad des Energieniveaus<br />

p ist.<br />

Es gilt also:<br />

H(λ) |Ψ(λ)〉 = E(λ) |Ψ(λ)〉 (19)<br />

wobei |Ψ(λ)〉 der Eigenzustand <strong>und</strong> E(λ) der Energieeigenwert zum<br />

gestörten Hamiltonoperator sind.<br />

Weiterhin wird angenommen, <strong>das</strong>s sowohl die Energieeigenwerte, als<br />

auch die Eigenfunktionen in Potenzen, von λ entwickelt werden können.<br />

<strong>und</strong><br />

E(λ) = ∑ i<br />

|Ψ(λ)〉 = ∑ i<br />

λ i ɛ i (20)<br />

λ i |i〉 (21)<br />

Setzt man all dies in die Schrödingergleichung ein <strong>und</strong> vergleicht sukzessive<br />

Terme nullter, erster, zweiter, etc. Ordnung in λ, so ergibt sich<br />

in a-ter Ordnung:<br />

(H 0 − ɛ 0 ) |a〉 + (V − ɛ 1 ) |a − 1〉 −<br />

a∑<br />

ɛ j |a − j〉 = 0 (22)<br />

j=2<br />

Fordert man zusätzlich, <strong>das</strong>s die Wellenfunktion gemäÿ<br />

〈Ψ(λ) |Ψ(λ)〉 = 1 (23)<br />

normiert ist, so kann man die Korrekturen in a-ter Ordnung berechnen.<br />

Für die Energie-Korrekturen ergibt sich:<br />

nullte Ordnung ɛ 0 = E 0 p<br />

erste Ordung ɛ 1 = 〈0| V |0〉<br />

zweite Ordung ɛ 2 = ∑ p≠n,i<br />

|〈ϕ i p|V |ϕ n〉| 2<br />

E 0 n−E 0 p<br />

2. Entarteter Ausgangszustand:<br />

Im entarteten Fall soll uns nun die Betrachtung der Energiekorrektur<br />

in erster Ordnung genügen. Die Betrachtung erfolgt analog zum nichtentarteten<br />

Fall.<br />

Der Gr<strong>und</strong>zustand sei gegeben durch:<br />

|0〉 =<br />

g n<br />

∑<br />

i=1<br />

∣<br />

c i ϕ i 〉<br />

n<br />

(24)<br />

13


wobei g n der Entartungsgrad ist.<br />

Führen wir jetzt einen Operator P 0 ein, der einen beliebigen Zustand<br />

in den Entartungsraum zur Energie n projiziert:<br />

P 0 =<br />

g n<br />

∑<br />

i=1<br />

∣<br />

∣ϕ i 〈 ∣<br />

n〉<br />

ϕ<br />

i<br />

n (25)<br />

Analog zum nicht-entarteten Fall erhält man in nullter <strong>und</strong> erster Ordnung<br />

in λ:<br />

H 0 |0〉 = E 0 n |0〉 (26)<br />

<strong>und</strong><br />

(H 0 − E 0 n) |1〉 + (V − ɛ 1 ) |0〉 = 0 (27)<br />

Wendet man P 0 auf Gleichung (27) an, so erhält man eine Eigenwertgleichung<br />

mit der sich die Energiekorrekturen bestimmen lassen. Formt<br />

man diese Gleichung dann noch unter Verwendung von<br />

<strong>und</strong> Gleichung (18) um, so erhält man:<br />

g n<br />

∑<br />

j=1<br />

P 0 |0〉 = |0〉 (28)<br />

〈 ∣<br />

ϕ<br />

i<br />

n V ∣ ∣ϕ j 〉<br />

n cj = ɛ i c i (29)<br />

Dies ist eine Eigenwertgleichung. Besitzt diese nicht g n gleiche Eigenwerte,<br />

so ergeben sich unterschiedliche Energiekorrekturen. D.h. die<br />

Zustände für die sich unterschiedliche Korrekturen ergeben sind nach<br />

hinzufügen der Störung nicht mehr entartet, sie spalten also auf. Je<br />

nach Symmetrie der Störung besteht auch die Möglichkeit, <strong>das</strong>s <strong>das</strong><br />

Energienieveau auch teilweise oder gar nicht aufspaltet. Mit Hilfe des<br />

<strong>Wigner</strong>-<strong>Eckart</strong>-<strong>Theorem</strong>s lässt sich dies (bis auf spezielle Ausnahmen)<br />

vorhersagen, ohne zuvor <strong>das</strong> Eigenwertproblem explizit zu lösen.<br />

2.6.2 Zeitabhängige Störungsrechnung<br />

Da wir uns im Folgenden insbesondere für Übergangswahrscheinlichkeiten<br />

zwischen diskreten Quantenzuständen interessieren, sollten wir uns zunächst<br />

damit befassen, wie man diese berechnet. Dazu bedienen wir uns der zeitabhängigen<br />

Störungsrechung.<br />

Wir betrachten ein nicht-entartetes, diskretes System von Quantenzuständen.<br />

Auch hier seien wieder die Eigenfunktionen <strong>und</strong> Eigenwerte des ungestörten<br />

Systems bekannt.<br />

H 0 |ϕ n 〉 = E n |ϕ n 〉 (30)<br />

14


Analog zum vorherigen Abschnitt fügen wir dem Hamiltonoperator eine nun<br />

zeitabhängige Störung Ṽ (t) = λV (t) hinzu. Es soll gelten:<br />

H(t) = H 0 + λV (t) mit λ ≪ 1 (31)<br />

Die zeitabhängige Störung soll zum Zeitpunkt t 0 einsetzen. D.h. V (t < t 0 ) =<br />

0. Als Anfangsbedingung nimmt man an, <strong>das</strong>s sich <strong>das</strong> System zum Zeitpunkt<br />

t 0 im Eigenzustand |ϕ i 〉 des ungestörten Systems bendet.<br />

|Ψ(t = t 0 )〉 = |ϕ i 〉 (32)<br />

Gemäÿ Postulat 6 ist die Wahrscheinlichkeit dafür, <strong>das</strong> System zum Zeitpunkt<br />

t im Zustand |ϕ f 〉 zu nden, gegeben durch:<br />

P fi (t) = |〈ϕ f |Ψ(t)〉| 2 (33)<br />

Unter unserer Anfangsbedingung ist dies genau die Wahrscheinlichkeit dafür,<br />

<strong>das</strong>s bis zum Zeitpunkt t der Übergang |ϕ i 〉 −→ |ϕ f 〉 stattgef<strong>und</strong>en hat.<br />

Diese Wahrscheinlichkeit wollen wir jetzt berechnen.<br />

Dazu entwickeln wir |Ψ(t)〉 nach dem zeitunabhängigen, vollständigen System<br />

von Eigenfunktionen des ungestörten Hamiltonoperators. Die Zeitabhängigkeit<br />

der Funktion ist daher nur in den Entwicklungskoezienten zu<br />

nden.<br />

|Ψ(t)〉 = ∑ c n (t) |ϕ n 〉 (34)<br />

n<br />

Des Weiteren wissen wir, <strong>das</strong>s die Zeitentwicklung der Eigenzustände eines<br />

zeitunabhängigen Hamiltonoperators folgende Form hat:<br />

En<br />

−i<br />

|Φ(t)〉 = e t |ϕ n 〉 (35)<br />

En<br />

−i<br />

Wenn wir also die Koezienten c n (t) durch c n (t) = e t b n (t) ersetzen, so<br />

können wir bei einer kleinen Störung annehmen, <strong>das</strong>s sich die b n (t), zumindest<br />

für kurze Zeiten, nur sehr wenig ändern.<br />

Setzt man die Entwicklung von |Ψ(t)〉 mit den Koezienten b n (t) <strong>und</strong> dem<br />

gestörten Hamiltonoperator in die zeitabhängige Schrödingergleichung ein<br />

<strong>und</strong> wendet von links 〈ϕ n | darauf an, so erhält man:<br />

i d dt b n(t) = λ ∑ m<br />

e iωmn 〈ϕ n | V (t) |ϕ m 〉 b m (t) (36)<br />

wobei ω mn = Em−En<br />

<br />

die Bohrfrequenz ist.<br />

Bis hier wurde noch nicht davon Gebrauch gemacht, <strong>das</strong>s die Störung klein<br />

ist. Diese Gleichung hier ist also noch äquivalent zur zeitabhängigen Schrödingergleichung.<br />

Nun wollen wir aber verwenden, <strong>das</strong>s sich die Koezienten<br />

15


n (t) nur langsam ändern. Ist dies der Fall, so lassen sich diese, für kurze<br />

Zeiten, in eine Potenzreihe in λ entwickeln:<br />

b n (t) = b (0)<br />

n (t) + b (1)<br />

n (t)λ + b (2)<br />

n (t)λ 2 + . . . (37)<br />

Setzt man diese Entwicklung ein, so erhält man in nullter Ordnung in λ:<br />

Mit der Anfangsbedingung folgt daraus:<br />

In erster Ordnung ergibt sich dann:<br />

i d dt b(1) n (t) = ∑ m<br />

i d dt b(0) n (t) = 0 (38)<br />

b (0)<br />

n (t) = δ ni (39)<br />

e iωnmt 〈ϕ n | V (t) |ϕ m 〉 b (0)<br />

m (t) (40)<br />

Verwendet man b (0)<br />

m (t) = δ mi <strong>und</strong> integriert die Gleichung von t 0 , was o.B.d.A<br />

gleich Null gesetzt werden kann, bis t, so ergibt sich:<br />

b (1)<br />

n (t) = 1<br />

i<br />

∫ t<br />

0<br />

e iω ni˜t 〈ϕ n | V (˜t) |ϕ i 〉d˜t (41)<br />

∣ ∣<br />

Für die Übergangswahrscheinlichkeit P if (t) = |〈ϕ f |Ψ(t)〉| 2 = λ 2 ∣∣b (1) ∣∣<br />

f (t) 2<br />

ergibt sich damit:<br />

∣∫ P if (t) = λ2 ∣∣∣ t<br />

2<br />

2 e iωfi˜t 〈ϕ f | V (˜t) |ϕ i 〉d˜t<br />

∣<br />

(42)<br />

0<br />

Im Falle einer sinusförmigen Störung, wie man sie bei der Absorption oder<br />

der stimulierten Emission von Photonen annehmen kann, ergibt sich:<br />

∣ P if (t) = |〈ϕ f | V |ϕ i 〉| 2 ∣∣∣∣<br />

e i(ωfi+ω)t − 1<br />

4 2 − ei(ωfi−ω)t − 1<br />

2<br />

(43)<br />

ω fi + ω ω fi − ω ∣<br />

Im Resonanzfall, also für ω → ±ω fi , kann man jeweils den einen Summanden<br />

zwischen den Betragsstrichen gegenüber dem Anderen vernachlässigen,<br />

dessen Nenner gegen Null geht 1 . Es ergibt sich dann:<br />

P if (t) = |〈ϕ f | V |ϕ i 〉| 2<br />

2<br />

t 2 ∣ ∣∣∣ sin(ω fi t/2)<br />

ω fi t/2<br />

∣<br />

2<br />

(44)<br />

Wie man hier sehr gut erkennen kann, ist die Übergangswahrscheinlichkeit<br />

zwischen zwei quantenmechanischen Zuständen proportional zum Betragsquadrat<br />

von 〈ϕ f | V |ϕ i 〉. Dies bezeichnet man als Übergangsmatrixelement.<br />

Insbesondere wird die Übergangswahrscheinlichkeit Null, wenn <strong>das</strong> Übergangsmatrixelement<br />

verschwindet.<br />

1<br />

Eine Begründung hierfür ist in Lehrbüchern der Quantenmechanik gegeben.<br />

16


3 Produkträume<br />

Besonders in der Quantenphysik hat man sehr häug mit Produkträumen<br />

zu tun. Beispielsweise setzt sich die Gesamtwellenfunktions eines Teilchens<br />

im Allgemeinen aus einer Funktion aus dem Ortsraum <strong>und</strong> einer Funktion<br />

aus dem Spinraum zusammen. Das gesamte System lässt sich dann in dem,<br />

durch <strong>das</strong> direkte Produkt von Orts- <strong>und</strong> Spinraum aufgespannten Raum,<br />

beschreiben. Diesen Produktraum kann man bezüglich der Darstellung einer<br />

Gruppe, über diesem Produktraum, in invariante Unterräume zerlegen. Wie<br />

dies umgesetzt wird, soll in diesem Abschnitt behandelt werden.<br />

Denition 1 (Direktes Produkt von Vektorräumen) Seien V <strong>und</strong> W<br />

Vektorräume mit den Basen {v1, . . . , vn} sowie {w1, . . . , wm}, dann bezeichnet<br />

man den, von allen direkten Produkten der Basisvektoren von V mit den<br />

Basisvektoren von W aufgespannten Vektorraum, als <strong>das</strong> direkte Produkt der<br />

Vektorräume V <strong>und</strong> W.<br />

V ⊗ W = 〈vi ⊗ wj|i = 1, . . . , n; j = 1, . . . , m〉<br />

Dies lässt sich ohne Weiteres auf <strong>das</strong> direkte Produkt von q Vektorräumen<br />

miteinander verallgemeinern.<br />

In der Darstellungstheorie lässt sich auÿerdem folgender Satz zeigen:<br />

Satz 2 (Satz zur Produktdarstellung) Seien D (α) : G → GL(V ) bzw.<br />

D (β) : G → GL(W ) unitäre, irreduzible Darstellungen der Gruppe G<br />

über dem Vektorraum V bzw. W mit den Basen {v (α)<br />

1 , . . . , v(α) d α<br />

} bzw.<br />

{u (β)<br />

1 , . . . , u(β) d β<br />

}, so ist die Produktdarstellung<br />

(α×β) Def.<br />

D = D (α) ⊗ D (β) : G → GL(V ⊗ W )<br />

eine im Allgemeinen reduzible Darstellung über dem Produktraum von V <strong>und</strong><br />

W. Auch dieser Satz lässt sich auf <strong>das</strong> Produkt von q unitären, irreduziblen<br />

Darstellungen verallgemeinern.<br />

Doch wie ergeben sich nun die Matrixelemente der Produktdarstellung aus<br />

denen der Darstellungen D (α) <strong>und</strong> D (β) ?<br />

17


Dazu betrachten <strong>das</strong> Transformationsverhalten eines beliebigen Vektors x<br />

aus V ⊗ W unter der Wirkung eines Gruppenelements g:<br />

(<br />

) ∑ (<br />

)<br />

¯x = D (α) (g) ⊗ D (β) (g) x = D (α) (g) ⊗ D (β) (g) x ij (v i ⊗ w j ) (45)<br />

i,j<br />

Die Darstellungen wirken jeweils nur auf den Teil des Basisvektors, der aus<br />

dem zur jeweiligen Darstellung gehörenden Vektorraum stammt.<br />

¯x = ∑ (<br />

)<br />

x ij D (α) (g)v i ⊗ D (β) (g)w j = ∑ (<br />

x ij D (α)<br />

ki (g)v k ⊗ D (β)<br />

lj<br />

(g)w )<br />

l<br />

i,j<br />

i,j,k,l<br />

(46)<br />

damit erhält man dann:<br />

¯x = ∑ ∑ (<br />

)<br />

D (α)<br />

ki (g)D(β) lj (g) x ij (v k ⊗ w l ) (47)<br />

k,l i,j<br />

} {{ }<br />

¯x kl<br />

Das heiÿt die Komponenten eines beliebigen Vektors aus dem Produktraum<br />

transformieren sich wie folgt:<br />

¯x kl = ∑ (<br />

)<br />

D (α)<br />

ki (g)D(β) lj (g) x ij = ∑ D (α×β)<br />

kl,ij<br />

(g)x ij (48)<br />

ij<br />

ij<br />

Man erhält also die Komponenten:<br />

D (α×β)<br />

kl,ij<br />

(g) = D (α)<br />

ki (g)D(β) lj<br />

(g) (49)<br />

Die Charaktere erhält man durch Spurbildung:<br />

χ (α×β) (g) = ∑ i,j<br />

D (α×β)<br />

ij,ij<br />

(g) = ∑ i,j<br />

D (α)<br />

ii<br />

(g)D (β) (g) (50)<br />

jj<br />

Die Summe lässt sich in zwei Faktoren zerlegen:<br />

{ } ⎧ ⎫<br />

∑ ⎨<br />

χ (α×β) (g) = D (α) ∑ ⎬<br />

ii<br />

(g) · D (β)<br />

⎩<br />

jj (g) ⎭ = χ(α) (g)χ (β) (g) (51)<br />

i<br />

j<br />

Es ergibt sich also:<br />

χ (α×β) (g) = χ (α) (g)χ (β) (g) (52)<br />

Die Charaktere der Produktdarstellung ergeben sich also als Produkt der<br />

Charaktere der einzelnen Darstellungen. Mit deren Hilfe lässt sich diese Produktdarstellung<br />

leicht in irreduzible Darstellungen zerlegen. Es gilt:<br />

D (α) ⊗ D (β) = ⊕ γ<br />

(αβ|γ)D (γ) (53)<br />

18


Dabei sind die Reduktionskoezienten<br />

(αβ|γ) = 1 ∑<br />

χ (α)∗ (g)χ (β)∗ (g)χ (γ) (g) (54)<br />

|G|<br />

g∈G<br />

die Multiplizitäten der irreduziblen Darstellungen in der Produktdarstellung.<br />

Um jetzt die Basisvektoren einer ausreduzierten (orthonormalen) Basis durch<br />

die Basisvektoren des Produktraumes auszudrücken, muss man einen Basiswechsel<br />

durchführen.<br />

= ∑<br />

u (γ,sγ)<br />

k<br />

Oder in Bra-Ket-Notation:<br />

α,i,β,j<br />

|γs γ k〉 = ∑<br />

α,i,β,j<br />

Für die Rücktransformation gilt:<br />

v (α)<br />

i<br />

In Bra-Ket-Notation:<br />

⊗ w (β)<br />

j<br />

v (α)<br />

i<br />

= ∑<br />

⊗ w (β)<br />

j<br />

〈αiβj |γs γ k〉 (55)<br />

|αi〉 ⊗ |βj〉 〈αiβj |γs γ k〉 (56)<br />

γ,s γ,k<br />

|α, i〉 ⊗ |β, j〉 = ∑<br />

γ,s γ,k<br />

u (γ,sγ)<br />

k<br />

〈γs γ k |αiβj〉 (57)<br />

|γ, s γ 〉 〈γs γ k |αiβj〉 (58)<br />

Dabei sind 〈αiβj |γs γ k〉 die Clebsch-Gordan-Koezienten (oder auch Kopplungskoezienten).<br />

Nebenbemerkung: Ermittlung der Clebsch-Gordan-Koezienten<br />

Mit Hilfe von Projektionsoperatoren (siehe Vortrag über Molekülschwingungen) lassen<br />

sich die invarianten Unterräume zu einer Darstellung nden. Wählt man in diesen invarianten<br />

Unterräumen, in standardisierter Art <strong>und</strong> Weise, (orthonormale) Basen, so sind<br />

die Koezienten in der Linearkombination, mit der die neuen irreduziblen Basisvektoren<br />

durch die Produktbasisvektoren ausgedrückt werden, die Clebsch-Gordan-Koezienten.<br />

Für wichtige Fälle sind die Clebsch-Gordan-Koezienten tabelliert.<br />

19


4 Tensoroperatoren<br />

4.1 Denition<br />

Viele physikalische Gröÿen sind unter der Wirkung einer Gruppe zwar nicht<br />

invariant, haben aber ein deniertes Transformationsverhalten, wie beispielsweise<br />

die Komponenten eines Ortsvektors unter der Wirkung von SO(3). Um<br />

später die Berechnung von Matrixelementen gruppentheoretisch vereinfachen<br />

zu können, müssen wir zunächst eben dieses Verhalten näher untersuchen.<br />

Denition 2 (Gruppenwirkung auf einen selbstadj. Operator) Sei<br />

T ein selbstadjungierter Operator im Vektorraum V (mit einem inneren<br />

Produkt 〈· |·〉). Sei auÿerdem G eine Gruppe mit längenerhaltender Wirkung<br />

auf V:<br />

G → GL(V )g ↦→ D(g) wobei D(g) † = D(g) −1 = D(g −1 )<br />

Dann ist der unter der Wirkung von G transformierte Operator T ′ , derjenige<br />

für den<br />

D(g)T = T ′ D(g) oder T ′ = D(g)T D(g) −1<br />

erfüllt ist.<br />

Damit ist eine Gruppenwirkung auf den Raum der selbstadjungierten Operatoren<br />

deniert. Man kann jetzt also die Methoden der Darstellungstheorie<br />

anwenden.<br />

Von groÿer Bedeutung für die Physik - besonders für die Quantenmechanik -<br />

sind die Tensoroperatoren, da die den Observablen zugehörigen Operatoren<br />

häug ein bestimmtes Transformationsverhalten besitzen.<br />

T (α)<br />

i<br />

{ Denition 3 (Satz} irreduzibler Tensoroperatoren) Die Menge<br />

, i = 1, . . . , d α bildet genau dann einen Satz irreduzibler Tensoroperatoren<br />

(zur irreduziblen Darstellung D (α) der Gruppe G), wenn sich<br />

ihre Elemente unter der Wirkung von G auf folgende Art <strong>und</strong> Weise<br />

transformieren:<br />

D(g)T (α)<br />

i<br />

D(g) −1 =<br />

d α ∑<br />

j=1<br />

D (α)<br />

ji<br />

(g)T (α)<br />

j<br />

20


Die irreduziblen Tensoroperatoren zur Darstellung D (α) transformieren sich<br />

also wie ein Satz von Basisfunktionen zu dieser Darstellung. Man kann also<br />

auch sagen: Ein Satz irreduzibler Tensoroperatoren zur Darstellung D (α)<br />

bildet eine Basis zur Darstellung D (α) .<br />

4.2 Beispiel: Ortsoperator<br />

Um mit dieser Denition vertraut zu werden wollen wir nun als Beispiel den<br />

Ortsoperator behandeln.<br />

Gesucht ist also <strong>das</strong> Transformationsverhalten des Ortsoperators unter der<br />

Wirkung von O(3).<br />

Dazu benötigen wir:<br />

• Transformationsverhalten eines Vektors unter O(3):<br />

[D(g)x] i<br />

=<br />

3∑<br />

j=1<br />

D (v)<br />

ij (g)x j (59)<br />

bzw. (60)<br />

[<br />

D(g −1 )x ] i<br />

=<br />

3∑<br />

D (v)<br />

ji (g)x j (61)<br />

j=1<br />

• Wirkung von g ∈ G auf eine skalare Funktion:<br />

(D(g)Ψ) (x 1 , x 2 , x 3 ) = Ψ ([ D(g) −1 x ] 1 , [ D(g) −1 x ] 2 , [ D(g) −1 x ] 3<br />

(62)<br />

D.h. die transformierte Funktion entspricht der unveränderten Funktion,<br />

ausgewertet am rücktransformierten Ort.<br />

• Die Wirkung des Ortsoperators auf eine Wellenfunktion ist in der Ortsdarstellung<br />

wie folgt deniert:<br />

ˆx i Ψ(x) = x i Ψ(x) (63)<br />

Gemäÿ Denition 2 gilt für den mit g transformierten Orstoperator ˆx ′ :<br />

D(g)ˆxΨ(x 1 , x 2 , x 3 ) = ˆx ′ D(g)Ψ(x 1 , x 2 , x 3 ) (64)<br />

Wir berechnen jetzt die linke Seite von Gleichung (64): Mit der Wirkung des<br />

Ortsoperators auf Wellenfunktionen ergibt sich:<br />

ˆx i Ψ(x 1 , x 2 , x 3 ) Def.<br />

= x i Ψ(x 1 , x 2 , x 3 ) (65)<br />

21<br />

)


Abbildung 2: Wirkung von c 6 auf die p-Wellenfunktionen der C-Atome eines<br />

Benzolmoleküls<br />

Setzt man weiter die Wirkung von g auf die skalare Funktion x i Ψ(x 1 , x 2 , x 3 )<br />

ein:<br />

= ( D(g −1 )x ) i Ψ ([ D(g) −1 x ] 1 , [ D(g) −1 x ] 2 , [ D(g) −1 x ] )<br />

(66)<br />

3<br />

Verwendet man <strong>das</strong> Transformationsverhalten von Ortsvektoren, so erhält<br />

man:<br />

3∑<br />

= D (v)<br />

ji (g)x j (D(g)Ψ(x 1 , x 2 , x 3 )) (67)<br />

j=1<br />

Mit der Denition des Ortsoperators ergibt sich schlieÿlich <strong>das</strong> Resultat:<br />

D(g)ˆx i Ψ(x 1 , x 2 , x 3 ) =<br />

3∑<br />

j=1<br />

Für den mit g transformierten Ortsoperator gilt:<br />

ˆx ′ i = D(g)ˆx i D(g) −1 =<br />

D (v)<br />

ji (g)ˆx j (D(g)Ψ(x 1 , x 2 , x 3 )) (68)<br />

3∑<br />

j=1<br />

D (v)<br />

ji (g)ˆx j (69)<br />

Die Komponenten des Ortsoperators transformieren sich also wie polare Basisvektoren<br />

zur Vektordarstellung. Sie bilden damit einen Satz irreduzibler<br />

Tensoroperatoren bezüglich D (v) .<br />

Weitere Beispiele für selbstadjungierte Operatoren sind:<br />

22


• Impulsoperator ˆp i Ψ(x 1 , x 2 , x 3 ) = −i ∂<br />

∂x i<br />

Ψ(x 1 , x 2 , x 3 ):<br />

ˆp ′ i = D(g)ˆp i D(g) −1 =<br />

3∑<br />

j=1<br />

D (v)<br />

ji (g)ˆp j (70)<br />

• Drehimpulsoperator ˆL i Ψ(x 1 , x 2 , x 3 ) = ɛ ijk ˆx j ˆp k Ψ(x 1 , x 2 , x 3 ):<br />

ˆL ′ i = D(g)ˆL i D(g) −1 =<br />

3∑<br />

j=1<br />

( )<br />

det D (v) (g) D (v)<br />

ji (g)ˆL j (71)<br />

Dabei transformiert sich die Komponenten des Impulsoperators unter O(3)<br />

gleich wie die des Ortsoperators, während sich die Komponenten des Drehimpulsoperators<br />

wie axiale Basisvektoren transformieren. D.h. gleich, bis auf<br />

den Faktor det ( D (v) (g) ) . Unter O(3) kann dieser Faktor die Werte ±1 annehmen.<br />

Enthält die Transformation Spiegelungen, so ist er -1, sonst 1. Physikalisch<br />

kann man sich diesen Sachverhalt an einem einfachen Beispiel klarmachen.<br />

Abbildung 3: Transformationsverhalten: axialer Vektor<br />

In Abbildung 3 ist ein Massepunkt dargestellt, der sich auf einer Kreisbahn<br />

bewegt. Der Drehimpulsvektor zeigt hier nach oben. Führt man eine Spiegelung<br />

an einer Ebene senkrecht zur Kreisbahn, durch den Kreismittelpunkt<br />

aus, so bewegt sich der Massepunkt im gespiegelten Problem in entgegengesetzter<br />

Richtung. D.h. der Drehimpulsvektor zeigt jetzt nach unten. Ein<br />

Ortsvektor wäre bei einer Spiegelung an einer Ebene, die ihn enthält, unverändert<br />

geblieben.<br />

23


5 <strong>Wigner</strong>-<strong>Eckart</strong>-<strong>Theorem</strong><br />

{<br />

}<br />

Satz 3 (<strong>Wigner</strong>-<strong>Eckart</strong>-<strong>Theorem</strong>) Sei T (α)<br />

i<br />

, i = 1, . . . , d α ein Satz<br />

irreduzibler Tensoroperatoren zur irreduziblen Darstellung D (α) <strong>und</strong><br />

{|β, j〉 , j = 1, . . . d β } bzw. {|γ, k〉 , k = 1, . . . d γ } ein Satz von Basisfunktionen<br />

zur irreduziblen Darstellung D (β) bzw. D (γ) . Dann gilt:<br />

〈<br />

ϕ (γ)<br />

k<br />

∣<br />

∣T (α)<br />

i<br />

〉<br />

∣ ϕ (β) =<br />

j<br />

(αβ|γ)<br />

∑<br />

s=1<br />

〈 ∥ ϕ (γ) ∥∥T (α)<br />

∥ ϕ (β)〉 〈γsk|αi, βj〉<br />

s<br />

mit den komplex konjugierten Clebsch-Gordan-Koezienten 〈γsk|αi, βj〉 <strong>und</strong><br />

dem reduzierten Matrixelement 〈 ϕ ∥ (γ) ∥T (α)∥ ∥ ϕ (β)〉 , <strong>das</strong> unabhängig von den<br />

s<br />

Spaltenindizes ist.<br />

Jetzt wollen wir <strong>das</strong> <strong>Wigner</strong>-<strong>Eckart</strong>-<strong>Theorem</strong> beweisen.<br />

Sei also { }<br />

T (α)<br />

i<br />

, i = 1, . . . , d α ein Satz irreduzibler Tensoroperatoren zur irreduziblen<br />

Darstellung D (α) <strong>und</strong> {∣ 〉<br />

}<br />

∣ (β) ∣ϕ j<br />

, j = 1, . . . d β ein Satz von Basisfunktionen<br />

zur irreduziblen Darstellung D (β) . Wir wollen 〉 uns zunächst<br />

∣ klarmachen. Dazu<br />

<strong>das</strong> Transformationsverhalten der Funktionen T (α)<br />

i<br />

lassen wir D(g) auf diese Funktionen wirken:<br />

D(g)T (α)<br />

i<br />

∣<br />

∣ϕ (β)<br />

j<br />

〉 (<br />

=<br />

D(g)T (α)<br />

i<br />

D(g) −1)<br />

} {{ }<br />

T ′(α)<br />

i<br />

∣ϕ (β)<br />

j<br />

(<br />

〉)<br />

∣<br />

D(g) ∣ϕ (β)<br />

j<br />

} {{ }<br />

〉<br />

∣<br />

∣ϕ ′(β)<br />

j<br />

(72)<br />

Setzt man <strong>das</strong> bekannte Transformationsverhalten der Tensoroperatoren <strong>und</strong><br />

der Basisfunktionen ein, so erhält man:<br />

(<br />

∣ 〉 dα<br />

) ⎛ d<br />

D(g)T i (α) ∣∣ϕ<br />

∑<br />

(β)<br />

j<br />

= D (α)<br />

∑ β ∣ 〉 ⎞ (α)<br />

ki<br />

(g)T ⎝<br />

∣∣ϕ<br />

k<br />

(g) (β)<br />

⎠ (73)<br />

k=1<br />

Damit ergibt sich letztendlich:<br />

〉<br />

D(g)T (α) ∣<br />

i ∣ϕ (β)<br />

j<br />

= ∑ k,l<br />

D (α)<br />

l=1<br />

ki (g)D(β) lj<br />

} {{ (g)<br />

}<br />

D (α×β)<br />

kl,ij (g)<br />

D (β)<br />

lj<br />

T (α)<br />

k<br />

∣<br />

l<br />

∣ϕ (β)<br />

l<br />

〉<br />

(74)<br />

Die betrachteten Funktionen transformieren sich also nach der Produktdar-<br />

〉<br />

stellung D (α) ⊗D (β) = D (α×β) . Anders formuliert: Die Funktionen T (α) ∣<br />

i ∣ϕ (β)<br />

j<br />

24


ilden einen Satz Basisfunktionen zur Produktdarstellung D (α×β) . Diese Produktdarstellung<br />

ist im Allgemeinen reduzibel. Wie oben beschrieben lässt<br />

sich die Produktdarstellung 〉 ausreduzieren, indem wir von der Produktbasis<br />

T (α) ∣<br />

i ∣ϕ (β)<br />

j in eine symmetrieangepasste Basis wechseln. Dies können wir mit<br />

Hilfe der Clebsch-Gordan-Koezienten bewerkstelligen:<br />

〉<br />

∣<br />

= ∑<br />

〉<br />

∣<br />

〈γs γ k |αiβj〉 (75)<br />

T (α)<br />

i<br />

∣ϕ (β)<br />

j<br />

∣Ψ (γ,sγ)<br />

k<br />

γ,s γ,k<br />

Auf diese Gleichung wollen wir nun von links 〈 ϕ (η)<br />

m<br />

sich: 〈 ∣ ∣ 〉<br />

= ∑<br />

ϕ (η)<br />

m<br />

∣ T (α) ∣<br />

i<br />

∣ϕ (β)<br />

j<br />

γ,s γ,k<br />

〈<br />

ϕ (η)<br />

m<br />

∣<br />

∣Ψ (γ,sγ)<br />

k<br />

∣ wirken lassen. Es ergibt<br />

〉<br />

〈γs γ k |αiβj〉 (76)<br />

Nun müssen wir noch <strong>das</strong> Skalarprodukt 〈 〉<br />

ϕ (η) ∣<br />

m ∣Ψ (γ,sγ)<br />

k berechnen: Dies soll<br />

hier nur für den Fall einer endlichen, diskreten Gruppe vorgerechnet werden.<br />

Die Rechnung lässt sich jedoch auf kompakte Gruppen verallgemeinern.<br />

Da D(g) eine unitäre Darstellung ist, gilt:<br />

〈<br />

〉 〈<br />

ϕ (η) ∣<br />

m ∣Ψ (γ,sγ) = D(g)ϕ (η) ∣<br />

m<br />

k<br />

∣D(g)Ψ (γ,sγ)<br />

k<br />

〉<br />

∀g ∈ G (77)<br />

Da dies für alle g ∈ G gilt <strong>und</strong> G endlich <strong>und</strong> diskret ist, lässt sich <strong>das</strong><br />

Skalarprodukt wie folgt schreiben:<br />

〈<br />

〉<br />

ϕ (η) ∣<br />

m ∣Ψ (γ,sγ)<br />

k<br />

= 1 ∑ 〈<br />

〉<br />

D(g)ϕ (η) ∣<br />

m ∣D(g)Ψ (γ,sγ)<br />

|G|<br />

k<br />

(78)<br />

g∈G<br />

Setzt man <strong>das</strong> bekannte Transformationsverhalten der Basisfunktionen ein,<br />

so ergibt sich:<br />

= 1 ∑<br />

|G|<br />

g∈G<br />

d γ<br />

∑<br />

d η<br />

∑<br />

i=1 j=1<br />

〈<br />

D (η)<br />

jm (g)ϕ(η) j<br />

Aus den Eigenschaften des Skalarproduktes folgt:<br />

= 1 ∑ ∑<br />

|G|<br />

g∈G<br />

i,j<br />

〈<br />

D (η)<br />

jm (g)∗ D (γ)<br />

ik (g) ϕ (η)<br />

j<br />

〉<br />

∣<br />

∣D (γ)<br />

ik (g)Ψ(γ,sγ) i<br />

∣<br />

∣Ψ (γ,sγ)<br />

i<br />

〉<br />

(79)<br />

(80)<br />

Umordnen liefert:<br />

= ∑ i,j<br />

〈<br />

ϕ (η) ∣<br />

j<br />

∣Ψ (γ,sγ)<br />

i<br />

〉 1 ∑<br />

|G|<br />

g∈G<br />

D (η)<br />

jm (g)∗ D (γ)<br />

ik (g)<br />

} {{ }<br />

1<br />

dη δηγδ jiδ mk<br />

(81)<br />

25


Mit der Orthogonalitätsrelation 1<br />

|G|<br />

folgt schlieÿlich:<br />

〈<br />

ϕ (η) ∣<br />

m<br />

∣Ψ (γ,sγ)<br />

k<br />

〉<br />

∑g∈G D(η) jm (g)∗ D (γ)<br />

ik (g) = 1<br />

d η<br />

δ ηγ δ ji δ mk<br />

= δ ηγ δ mk<br />

1<br />

d η<br />

∑<br />

〈<br />

ϕ (η)<br />

j<br />

Setzt man dies in Gleichung (76) ein, so ergibt sich:<br />

〈<br />

ϕ (η)<br />

k<br />

∣<br />

∣T (α)<br />

i<br />

〉<br />

∣ ϕ (β) =<br />

j<br />

(αβ|η)<br />

∑<br />

s=1<br />

∣<br />

∣Ψ (γ,sγ)<br />

j<br />

(82)<br />

j<br />

} {{ }<br />

Def.<br />

= K (γ,sγ )<br />

K (γ,sγ) 〈ηsk|αi, βj〉 (83)<br />

Deniert man schlieÿlich K (γ,sγ) = 〈 ϕ ∥ (η) ∥T (α)∥ ∥ ϕ (β)〉 als reduziertes Matrixelement,<br />

so erhält man <strong>das</strong><br />

s<br />

<strong>Wigner</strong>-<strong>Eckart</strong>-<strong>Theorem</strong>:<br />

〈<br />

ϕ (η)<br />

k<br />

∣<br />

∣T (α)<br />

i<br />

〉<br />

∣ ϕ (β) =<br />

j<br />

(αβ|η)<br />

∑<br />

s=1<br />

〈 ∥ ϕ (η) ∥∥T (α)<br />

∥ ϕ (β)〉 〈ηsk|αi, βj〉 (84)<br />

s<br />

Durch <strong>das</strong> <strong>Wigner</strong>-<strong>Eckart</strong>-<strong>Theorem</strong> wird die Berechnung von Matrixelementen<br />

in zwei Teile separiert. Nämlich in die Berechnung des reduzierten Matrixelements,<br />

<strong>das</strong> nicht mehr von den Spaltenindizes abhängt, <strong>und</strong> in die<br />

Ermittlung der Clebsch-Gordan-Koenienzen. Diese werden von der Symmetrie<br />

des Problems bestimmt. Zur Berechnung der reduzierten Matrixelemente<br />

muss man jedoch die genauen Wellenfunktionen kennen. D.h. man benötigt<br />

die vollständige physikalische Information. Mit Hilfe dieses <strong>Theorem</strong>s<br />

kann man also allein durch Betrachtung der Symmetrie Aussagen darüber<br />

treen, ob die Matrixelemente verschwinden oder nicht (→ <strong>Auswahlregeln</strong>).<br />

Des Weiteren erlaubt uns <strong>das</strong> <strong>Theorem</strong>, bei der Betrachtung von bestimmten<br />

Übergängen 2 , Aussagen über die Intensitätsverhältnisse zwischen diesen<br />

Übergängen zu treen.<br />

Insbesondere verschwinden die Clebsch-Gordan-Koezienten 〈ηsk|αi, βj〉 <strong>und</strong><br />

damit die Matrixelemente immer, wenn die irreduzible Darstellung D (η) nicht<br />

in der Produktdarstellung D (α) ⊗ D (β) enthalten ist.<br />

2<br />

Solche deren Anfangszustände zu den gleichen irreduziblen Darstellungen gehören <strong>und</strong><br />

ebenso deren Endzustände. Dann ist <strong>das</strong> reduzierte Matrixelement für alle gleich.<br />

〉<br />

26


6 Physikalische Anwendungen des <strong>Wigner</strong>-<strong>Eckart</strong>-<br />

<strong>Theorem</strong>s<br />

In diesem Abschnitt wollen wir uns nun um spezielle Anwendungen des<br />

<strong>Wigner</strong>-<strong>Eckart</strong>-<strong>Theorem</strong>s in der Physik kümmern.<br />

6.1 <strong>Auswahlregeln</strong><br />

Eine besonders wichtige Anwendung des <strong>Wigner</strong>-<strong>Eckart</strong>-<strong>Theorem</strong>s ist die Ermittlung<br />

von <strong>Auswahlregeln</strong> verschiedener Arten von Übergängen in quantenmechanischen<br />

Systemen.<br />

6.1.1 Elektromagnetische Dipolübergänge<br />

Wichtige Vertreter solcher Übergänge sind die elektrischen Dipolübergänge.<br />

Wie wir in Abschnitt 2.6 gesehen haben, ist die Wahrscheinlichkeit (in erster<br />

Ordnung), für Übergänge zwischen den Zuständen |ϕ i 〉 <strong>und</strong> |ϕ f 〉, proportional<br />

zum Betragsquadrat des Matrixelements 〈ϕ f |V | ϕ i 〉. Für einen elektrischen<br />

Dipolübergang wird die Störung durch eine elektromagnetische Welle<br />

verursacht. Der zugehörige Störoperator kann auf folgende Art <strong>und</strong> Weise<br />

dargestellt werden:<br />

V = − e mc p · A = − e mc p · [<br />

A0 e i(k·x−ωt) + A ∗ 0e −i(k·x−ωt)] (85)<br />

Dabei ist A <strong>das</strong> Vektorpotential des zeitabhängigen elektrischen Feldes, p<br />

der Impuls-, x der Ortsoperator <strong>und</strong> k der Wellenvektor.<br />

Es ist k·x ≪ 1, da sich 〈|x|〉 a 0 in der Gröÿenordnung des Bohrschen Radius<br />

a 0 <strong>und</strong> λ = 2π<br />

|k| ≫ a 0 für die betrachteten Wellenlängenbereiche (z.B. für den<br />

optischen Bereich) wesentlich gröÿer als der Bohrsche Radius ist, lässt sich<br />

der ortsabhängige Teil der Exponentialfunktion als konstant ansehen. Mit<br />

A 0 = − i E 2ω 0 ergibt sich also näherungsweise für den Störoperator:<br />

V =<br />

e<br />

mωc p · E 0sin(ωt) (86)<br />

verwendet man zusätzlich, <strong>das</strong>s p = m i [x, H 0] gilt, so ergibt sich für <strong>das</strong><br />

Matrixelement:<br />

〈ϕ f |[x, H 0 ]| ϕ i 〉 = − ie<br />

ωc E 0sin(ωt)(E i − E f ) 〈ϕ f |x| ϕ i 〉 ∝ e 〈ϕ f |x| ϕ i 〉 · E<br />

(87)<br />

Zieht man die Konstante e in <strong>das</strong> Matrixelement hinein, so steht der Dipoloperator<br />

d = ex in der Mitte. ( → Dipolübergang)<br />

27


〈ϕ f |[x, H 0 ]| ϕ i 〉 = 〈ϕ f |d| ϕ i 〉 · E (88)<br />

Um die <strong>Auswahlregeln</strong> unter einer bestimmten Symmetriegruppe zu bestimmen,<br />

benötigen wir jetzt nur noch <strong>das</strong> Transformationsverhalten des Ortsbzw.<br />

Dioploperators.<br />

1. Symmetriegruppe O(3):<br />

Wir wollen also herausnden, wann <strong>das</strong> Matrixelement 〈ϕ f |d i | ϕ i 〉, unter<br />

O(3) Symmetrie, verschwindet. Die irreduziblen Darstellungen der<br />

Gruppe O(3) lassen sich durch die Drehimpulsquantenzahlen charakterisieren.<br />

Die Kugelächenfunktionen Y l m bilden zu jedem l eine Basis<br />

zum zugehörigen invarianten Unterraum der irreduziblen Darstellung.<br />

Unter der Wirkung von O(3) transformiert sich der Ortsoperator <strong>und</strong><br />

damit auch der Dipoloperator nach den Spalten der irreduziblen Darstellung<br />

D (l=1) (g).<br />

Die quantenmechanischen Zustände lassen sich ebenfalls nach den irreduziblen<br />

Darstellungen der Symmetriegruppe einteilen. Sie transformieren<br />

sich also auch nach den Drehimpulsdarstellungen D (l) (g). Für<br />

die Produktdarstellung D (1) (g)⊗D (l) (g) ergibt sich folgende Zerlegung<br />

in irreduzible Darstellungen:<br />

D (1) (g) ⊗ D (l) (g) = D (l−1) (g) ⊕ D (l) (g) ⊕ D (l+1) (g) (89)<br />

Da nach dem <strong>Wigner</strong>-<strong>Eckart</strong>-<strong>Theorem</strong> nur solche Matrixelemente ungleich<br />

Null sein können, bei denen die, dem Endzustand zugehörige irreduzible<br />

Darstellung, in der Produktdarstellung von Anfangszustand<br />

<strong>und</strong> Operator enthalten ist, können wir schon jetzt alle Übergänge ausschlieÿen,<br />

bei denen sich die Quantenzahl l um mehr als 1 ändert.<br />

Weiterhin ist bekannt, <strong>das</strong>s der Ausgangszustand |ϕ i 〉 die Parität (−1) l<br />

<strong>und</strong> der Ortsoperator die Parität -1 hat. Da sich x j |ϕ i 〉 nach der<br />

Produktdarstellung transformiert, müssen die sich ergebenden, irreduziblen<br />

Darstellungen alle die Parität (−1) (l+1) haben. Der Endzustand<br />

kann jedoch nicht gleichzeitig zur irreduziblen Darstellung D (l) (g) gehören<br />

<strong>und</strong> die Parität (−1) (l+1) haben. Der Übergang l → l ist also<br />

nicht möglich. Deshalb reicht es im Folgenden aus, die irreduziblen<br />

Darstellungen D (l−1) <strong>und</strong> D (l+1) zu betrachten.<br />

Aus diesen beiden Betrachtungen erhalten wir daher die Auswahlregel<br />

für die Quantenzahl l bei Dipolübergängen.<br />

28


Satz 4 (Erste Auswahlregel für elektrische Dipolübergänge) Unter<br />

O(3) Symmetrie sind elektrische Dipolübergänge nur dann möglich, wenn<br />

sich die Drehimpulsquantenzahl l um plus oder minus 1 ändert. D.h. es sind<br />

nur Übergänge der Form l −→ l ± 1 erlaubt.<br />

Bisher haben wir nur untersucht, welche irreduziblen Darstellungen in<br />

der Produktdarstellung von Ortsoperator <strong>und</strong> Anfangszustand enthalten<br />

sind <strong>und</strong> welche dieser Darstellungen mit der Parität der Endzustände<br />

verträglich sind. So kann man durch eine relativ einfache <strong>und</strong><br />

schnelle Betrachtung erste Schlüsse aus den Symmetrieeigenschaften<br />

ziehen. Durch eine eingehendere Betrachtung können wir jedoch noch<br />

mehr über die Matrixelemente erfahren. Dazu müssen wir uns die Eigenschaften<br />

der Kopplungskoezienten etwas näher anschauen.<br />

Allgemein gilt für die Kopplungskoezienten 〈l 3 , m 3 |l 2 , m 2 ; l 1 , m 1 〉 unter<br />

O(3)-Symmetrie, <strong>das</strong>s sie nur von Null verschieden sind, wenn<br />

|l 2 − l 1 | ≤ l 3 ≤ l 1 + l 2 <strong>und</strong> zusätzlich m 1 + m 2 = m 3 erfüllt ist. Da<br />

m l ∈ {−l, −l + 1, . . . , l}, erhält man m 3 ∈ {−(l 1 + l 2 ), . . . , l 1 + l 2 }.<br />

Verwenden wir nun <strong>das</strong> <strong>Wigner</strong>-<strong>Eckart</strong>-<strong>Theorem</strong>, um die Matrixelemente<br />

zu berechnen. Für diesen Fall nimmt es folgende Form an:<br />

〈<br />

l f , m f<br />

∣ ∣∣d (l=1)<br />

j<br />

〉 ∣ l i , m i =<br />

(1l i |l f)<br />

∑<br />

s=1<br />

〈 ∥ ∥∥d (l=1)<br />

l f ∥ l i<br />

〉s 〈l f , m f |1j, l i m i 〉 (90)<br />

Da die möglichen irreduziblen Darstellungen des Endzustandes höchstens<br />

einmal in der Produktdarstellung, aus der irreduziblen Darstellung<br />

zum Ortsoperator <strong>und</strong> der zum Anfangszustand, enthalten ist,<br />

entfällt die Summe über die Multiplizitäten. Da die Clebsch-Gordan-<br />

Koezienten für m l=1 + m i ≠ m f verschwinden <strong>und</strong> m l=1 nur die<br />

Werte -1,0,1 annehmen kann, folgt eine zweite Auswahlregel:<br />

Satz 5 (Zweite Auswahlregel für elektrische Dipolübergänge)<br />

Unter O(3) Symmetrie sind elektrische Dipolübergänge nur dann möglich,<br />

wenn sich die magnetische Quantenzahl m l um 0, plus oder minus 1 ändert.<br />

D.h. es sind nur Übergänge der Form m l −→ m l , m l ± 1 erlaubt.<br />

Es ergeben sich also für elektrische Dipolübergänge unter O(3)-Symmetrie<br />

folgende, von Null verschiedene, Matrixelemente:<br />

29


〈<br />

∣<br />

l i − 1, m i − 1<br />

∣d (l=1)<br />

−1<br />

〉<br />

∣ l i, m i<br />

〈 ∣ 〉<br />

∣∣d (l=1)<br />

l i − 1, m i ∣ l i, m i<br />

0<br />

〈<br />

∣<br />

l i − 1, m i + 1<br />

〈<br />

∣<br />

l i + 1, m i − 1<br />

∣d (l=1)<br />

1<br />

∣d (l=1)<br />

−1<br />

〉<br />

∣ l i, m i<br />

〉<br />

∣ l i, m i<br />

=<br />

=<br />

=<br />

⎧<br />

⎪⎨<br />

⎪⎩<br />

〈<br />

∥ 〉<br />

∥<br />

l i − 1 ∥d (l=1) ∥∥<br />

li 〈l i − 1, m i − 1 |1, −1; l i, m i〉<br />

falls |m i − 1| ≤ l i − 1<br />

0 sonst<br />

⎧ 〈<br />

∥ 〉<br />

∥<br />

⎪⎨ l i − 1 ∥d (l=1) ∥∥<br />

li 〈l i − 1, m i |1, 0; l i, m i〉<br />

falls |m i| ≤ l i − 1<br />

⎪⎩<br />

0 sonst<br />

⎧<br />

⎪⎨<br />

⎪⎩<br />

〈<br />

∥ 〉<br />

∥<br />

l i − 1 ∥d (l=1) ∥∥<br />

li 〈l i − 1, m i + 1 |1, +1; l i, m i〉<br />

falls |m i + 1| ≤ l i − 1<br />

0 sonst<br />

= 〈 ∥ 〉<br />

∥<br />

l i + 1 ∥d (l=1) ∥∥<br />

li 〈l i + 1, m i − 1 |1, −1; l i, m i〉<br />

〈 ∣ 〉<br />

∣∣d (l=1)<br />

l i + 1, m i ∣ l i, m i<br />

0<br />

= 〈 ∥ 〉<br />

∥<br />

l i + 1 ∥d (l=1) ∥∥<br />

li 〈l i + 1, m i |1, 0; l i, m i〉<br />

〈<br />

∣<br />

l i + 1, m i + 1<br />

∣d (l=1)<br />

1<br />

〉<br />

∣ l i, m i<br />

= 〈 ∥ 〉<br />

∥<br />

l i + 1 ∥d (l=1) ∥∥<br />

li 〈l i + 1, m i + 1 |1, +1; l i, m i〉<br />

Wir sehen hier, <strong>das</strong>s alle Matrixelemente <strong>und</strong> damit die Übergangswahrscheinlichkeiten<br />

immer verschwinden, wenn mindestens einer der<br />

beiden <strong>Auswahlregeln</strong> nicht erfüllt ist. Auÿerdem sehen wir, <strong>das</strong>s Übergänge<br />

mit ∆m l = ±1 von den Komponenten d (l=1)<br />

±1 verursacht werden.<br />

Physikalisch entspricht dies einer Störung mit zirkular in der x-y-Ebene<br />

polarisiertem Licht. Übergänge mit ∆m = 0 hingegen werden von der<br />

Komponente d (l=1)<br />

0 verursacht. Dies entspricht einer Störung mit linear<br />

in z-Richtung polarisiertem Licht. Auch die Auswahlregel |∆l| = 1<br />

kann physikalisch gedeutet werden. Sie spiegelt die Drehimpulserhaltung<br />

wider. Denn ein Photon besitzt den Drehimpuls <strong>und</strong> wenn sich<br />

die Drehimpulsquantenzahl um 1 ändert, ändert sich der Bahndrehimpuls<br />

damit auch um .<br />

Für <strong>das</strong> konkrete Beispiel l i = 1 <strong>und</strong> m i = 1, also für einen p-Zustand<br />

mit L z -Komponente m i = , ergeben sich also die Matrixelemente:<br />

〈<br />

∣<br />

0, 0<br />

〈<br />

∣<br />

2, 0<br />

〈<br />

∣<br />

2, 1<br />

〈<br />

∣<br />

2, 2<br />

∣d (l=1)<br />

−1<br />

∣d (l=1)<br />

−1<br />

∣d (l=1)<br />

0<br />

∣d (l=1)<br />

1<br />

〉<br />

∣ 1, 1<br />

〉<br />

∣ 1, 1<br />

〉<br />

∣ 1, 1<br />

〉<br />

∣ 1, 1<br />

= 〈 ∥ 〉<br />

∥<br />

0 ∥d (l=1) ∥∥ 1 〈0, 0 |1, −1; 1, 1〉 = 〈 ∥ 〉 √<br />

∥<br />

0 ∥d (l=1) ∥∥ 1<br />

= 〈 ∥ 〉<br />

∥<br />

2 ∥d (l=1) ∥∥ 1 〈2, 0 |1, −1; 1, 1〉 = 〈 ∥ 〉 √<br />

∥<br />

2 ∥d (l=1) ∥∥ 1<br />

= 〈 ∥ 〉<br />

∥<br />

2 ∥d (l=1) ∥∥ 1 〈2, 1 |1, 0; 1, 1〉 = 〈 ∥ 〉 √<br />

∥<br />

2 ∥d (l=1) ∥∥ 1<br />

= 〈 ∥ 〉<br />

∥<br />

2 ∥d (l=1) ∥∥ 1 〈2, 2 |1, +1; 1, 1〉 = 〈 ∥ 〉<br />

∥<br />

2 ∥d (l=1) ∥∥ 1 1<br />

1<br />

3<br />

1<br />

6<br />

1<br />

2<br />

30


Vom p-Zustand mit m = 1 sind also nur Dipolübergänge in den s-<br />

Zustand <strong>und</strong> die d-Zustände mit m = 0, 1, 2 möglich. Man kann aus<br />

diesem Ergebnis schlieÿlich noch die Verhältnisse der Übergangswahrscheinlichkeiten<br />

in die Zustände mit l=2 ablesen. Diese verhalten sich<br />

so, wie die Quadrate der Kopplungskoezienten. Um die absoluten<br />

Übergangswahrscheinlichkeiten zu erhalten, muss man schlieÿlich noch<br />

die reduzierten Matrixelemente berechnen <strong>und</strong> dann <strong>das</strong> Ergebnis in<br />

die oben angegebene Gleichung für die Übergangswahrscheinlichkeiten<br />

einsetzen.<br />

2. Symmetriegruppe D 4h :<br />

Jetzt wollen wir uns die Dipolmatrixelemente für eine niedrigere Symmetrie,<br />

nämlich D 4h , anschauen. Dazu benötigen wir die Charaktertafel<br />

der Gruppe D 4h <strong>und</strong> auÿerdem die Charaktere der nun reduziblen<br />

Darstellung des Ortsoperators D (l=1) (g).<br />

Zunächst betrachten wir die Charaktere der Darstellung D (l=1) (g).<br />

Eine Drehung um einen Winkel ϕ, um die Achse parallel zu n, lässt sich<br />

bei geeigneter Wahl der Basis (symmetrieangepasste Basis) in folgender<br />

Form darstellen:<br />

⎛<br />

D(R ϕ,n ) = ⎝<br />

e iϕ 0 0<br />

0 e −iϕ 0<br />

0 0 1<br />

Der Charakter, also die Spur dieser Darstellung, ist<br />

⎞<br />

⎠ (91)<br />

χ(R ϕ,n ) = 1 + e iϕ + e −iϕ = 1 + 2cos(ϕ) (92)<br />

Da sich die Spur unter Konjugation mit anderen Gruppenelementen<br />

nicht verändert, haben wir hier den Charakter für Drehungen um den<br />

Winkel ϕ bestimmt. Enthält die Drehung noch zusätzlich eine Spiegelung<br />

an einer Ebene mit Normalenvektor n, so kann die Rotationsachse<br />

orthogonal zur Spiegelebene gewählt werden <strong>und</strong> es ändert sich in der<br />

ausreduzierten Darstellung nur der Teil entlang der Rotationsachse.<br />

Die Darstellung hat dann die Form:<br />

⎛<br />

D(S ϕ,n ) = ⎝<br />

Für den Charakter ergibt sich also<br />

e iϕ 0 0<br />

0 e −iϕ 0<br />

0 0 −1<br />

⎞<br />

⎠ (93)<br />

χ(S ϕ,n ) = −1 + e iϕ + e −iϕ = −1 + 2cos(ϕ) (94)<br />

Zusätzlich benötigen wir noch die Charaktertafel von D 4h . Diese kann<br />

man der Literatur entnehmen:<br />

31


D 4h E 2 C 4 C 2 2C 2 ′ 2 C 2 ′′ i 2 S 4 σ h 2 σ v 2 σ d<br />

A 1g 1 1 1 1 1 1 1 1 1 1<br />

A 2g 1 1 1 -1 -1 1 1 1 -1 -1<br />

A 1u 1 1 1 1 1 -1 -1 -1 -1 -1<br />

A 2u 1 1 1 -1 -1 -1 -1 -1 1 1<br />

B 1g 1 -1 1 1 -1 1 -1 1 1 -1<br />

B 2g 1 -1 1 -1 1 1 -1 1 -1 1<br />

B 1u 1 -1 1 1 -1 -1 1 -1 -1 1<br />

B 2u 1 -1 1 -1 1 -1 1 -1 1 -1<br />

E g 2 0 -2 0 0 2 0 -2 0 0<br />

E u 2 0 -2 0 0 -2 0 2 0 0<br />

Jetzt müssen wir die Charaktere der nun reduziblen Darstellung D (l=1) (g)<br />

bzgl. der Konjugationsklassen von D 4h bestimmen.<br />

D 4h E 2 C 4 C 2 2C ′ 2 2 C ′′<br />

2 i 2 S 4 σ h 2 σ v 2 σ d<br />

D (l=1) | D4h 3 1 -1 -1 -1 -3 -1 1 1 1<br />

Damit zerfällt D (l=1) (g), unter D 4h , in die irreduziblen Darstellungen<br />

A 2u <strong>und</strong> E u .<br />

D (l=1) | D4h = A 2u ⊕ E u (95)<br />

Da die irreduziblen Darstellungen, nach denen sich der Orts- bzw. Dipoloperator<br />

transformiert, ausschlieÿlich ungerade Parität haben, müssen<br />

auch hier alle Matrixelemente verschwinden, deren Anfangs- <strong>und</strong><br />

Endzustände gleiche Parität haben.<br />

Die zu den irreduziblen Darstellungen gehörenden Basisfunktionen sind:<br />

A 2u : d z = d (A 2u)<br />

{<br />

⇋ u(A 2u )<br />

d (Eu)<br />

x ⇋ u<br />

E u :<br />

x (E u )<br />

d (Eu)<br />

y<br />

⇋ u y (E u )<br />

In diesem Beispiel wollen wir einen Anfangszustand betrachten, der<br />

zur irreduziblen Darstellung E u gehört:<br />

|ϕ i 〉 = |ϕ(E u , j)〉<br />

Welche Endzustände sind von diesem Zustand aus erreichbar?<br />

Dazu ist es sinnvoll sich zunächst zu überlegen, nach welchen irreduziblen<br />

Darstellungen sich die möglichen Endzustände überhaupt transformieren<br />

können. Denn nach dem <strong>Wigner</strong>-<strong>Eckart</strong>-<strong>Theorem</strong> sind solche<br />

Endzustände, die nicht in der Produktdarstellung aus der irreduziblen<br />

32


Darstellung des Anfangszustandes <strong>und</strong> der des Dipoloperators auftauchen,<br />

verboten. Wir benötigen also die irreduziblen Darstellungen der<br />

Produktdarstellung:<br />

D (l=1) (g) ⊗ E u = (A 2u ⊕ E u ) ⊗ E u = (A 2u ⊗ E u ) ⊕ (E u ⊗ E u ) (96)<br />

Die Ergebnisse dieser Produkte sind (z.B. in [1]) tabelliert. Es ergibt<br />

sich:<br />

D (l=1) (g) ⊗ E u = E g ⊕ A 1g ⊕ A 2g ⊕ B 1g ⊕ B 2g (97)<br />

}{{} } {{ }<br />

A 2u ⊗E u E u⊗E u<br />

Es sind also von E u aus nur die Zustände A 1g , A 2g , B 1g , B 2g , E g erreichbar.<br />

Diese sind, wie schon oben erwähnt, ausschlieÿlich Zustände<br />

gerader Parität.<br />

Jetzt sollen die Matrixelemente, bis auf die reduzierten Matrixelemente,<br />

berechnet werden. Da die Anteile A 1g , A 2g , B 1g , B 2g allein von<br />

E u ⊗ E u erzeugt werden, benötigen wir zur Berechnung der zugehörigen<br />

Matrixelemente die Kopplungskoezienten für {u x (E u ), u y (E u )}⊗<br />

{v x (E u ), v y (E u )} in D 4h -Symmetrie. Wobei u x (E u ) <strong>und</strong> u y (E u ) die<br />

Basisfunktionen sind, die mit den Komponenten des Orts- bzw. Dipoloperators<br />

zur irreduziblen Darstellung E u indentiziert werden. v x (E u )<br />

<strong>und</strong> v y E u sind die Basisfunktionen, die zum Anfangszustand zur irreduziblen<br />

Darstellung E u gehören.<br />

Die Kopplungskoezienten kann man folgender Tabelle entnehmen:<br />

D 4h u x (E u )v x (E u ) u x (E u )v y (E u ) u y (E u )v x (E u ) u y (E u )v y (E u )<br />

w(A 1g ) √2 1<br />

0 0 √2 1<br />

w(A 2g ) 0 √2 1<br />

− √ 1<br />

2<br />

0<br />

w(B 1g ) √2 1<br />

0 0 − √ 1<br />

2<br />

w(B 2g ) 0 √2 1<br />

1<br />

√2 0<br />

Nach dem <strong>Wigner</strong>-<strong>Eckart</strong>-<strong>Theorem</strong> ergeben sich also zum Anfangszustand<br />

E u folgende Matrixelemente:<br />

33


〈<br />

ϕ(A1g ) ∣ ∣d (A 2u) ∣ ∣ ϕ(E u , j) 〉 = 0<br />

〈<br />

∣<br />

ϕ(A 1g )<br />

〈<br />

∣<br />

ϕ(A 1g )<br />

∣d (Eu)<br />

x<br />

∣d (Eu)<br />

y<br />

〉 〈 ∥<br />

∣ ϕ(E u , j) = √2 1<br />

δ ∥ 〉<br />

jx A1g ∥d (E u) A1g<br />

〉 〈 ∥<br />

∣ ϕ(E u , j) = √2 1<br />

δ jy A1g d (Eu)∥ 〉<br />

∥ A 1g<br />

〈<br />

ϕ(A2g ) ∣ ∣d (A 2u) ∣ ∣ ϕ(E u , j) 〉 = 0<br />

〈<br />

∣<br />

ϕ(A 2g )<br />

〈<br />

∣<br />

ϕ(A 2g )<br />

∣d (Eu)<br />

x<br />

∣d (Eu)<br />

y<br />

〉 〈 ∥<br />

∣ ϕ(E u , j) = √2 1<br />

δ jy A1g d (Eu)∥ 〉<br />

∥ A2g<br />

〉 〈 ∥<br />

∣ ϕ(E u , j) = − √2 1<br />

δ jx A1g d (Eu)∥ 〉<br />

∥ A 2g<br />

〈<br />

ϕ(B1g ) ∣ ∣d (A 2u) ∣ ∣ ϕ(E u , j) 〉 = 0<br />

〈<br />

∣<br />

ϕ(B 1g )<br />

〈<br />

∣<br />

ϕ(B 1g )<br />

∣d (Eu)<br />

x<br />

∣d (Eu)<br />

y<br />

〉 〈 ∥<br />

∣ ϕ(E u , j) = √2 1<br />

δ jx B1g d (Eu)∥ 〉<br />

∥ B 1g<br />

〉 〈 ∥<br />

∣ ϕ(E u , j) = − √2 1<br />

δ jy B1g d (Eu)∥ 〉<br />

∥ B 1g<br />

〈<br />

ϕ(B2g ) ∣ ∣ d<br />

(A 2u ) ∣ ∣ ϕ(Eu , j) 〉 = 0<br />

〈<br />

∣<br />

ϕ(B 2g )<br />

〈<br />

∣<br />

ϕ(B 2g )<br />

∣d (Eu)<br />

x<br />

∣d (Eu)<br />

y<br />

〉 〈 ∥<br />

∣ ϕ(E u , j) = √2 1<br />

δ jy B2g d (Eu)∥ 〉<br />

∥ B 2g<br />

〉 〈 ∥<br />

∣ ϕ(E u , j) = √2 1<br />

δ jx B2g d (Eu)∥ 〉<br />

∥ B 2g<br />

Durch <strong>das</strong> <strong>Wigner</strong>-<strong>Eckart</strong>-<strong>Theorem</strong> wurde also die Berechnung von<br />

insgesamt 24 Matrixelementen (48 wenn man die Matrixelemente mit<br />

Endzuständen ungerader Parität auch mitzählt), auf die Berechnung<br />

von 4 verschiedenen reduzierten Matrixelementen, vereinfacht.<br />

Um nun schlieÿlich noch die Matrixelemente mit Endzuständen zu berechnen,<br />

die sich nach E g transformieren, benötigen wir die Kopplungskoezienten<br />

für {u(A 2u )}⊗{v x (E u ), v y (E u )} in D 4h -Symmetrie. Denn<br />

die irreduzible Darstellung E g ergibt sich aus A 2u ⊗ E u .<br />

Diese sind in folgender Tabelle dargestellt:<br />

D 4h u(A 2u )v x (E u ) u(A 2u )v y (E u )<br />

w x (E g ) 0 1<br />

w y (E g ) -1 0<br />

Damit ergeben sich folgende Matrixelemente:<br />

34


〈<br />

ϕ(Eg , x) ∣ ∣d (A 2u) ∣ ∣ ϕ(E u , x) 〉 = 0<br />

〈<br />

ϕ(Eg , y) ∣ ∣d (A 2u) ∣ ∣ ϕ(Eu , x) 〉 = −1 〈 E g<br />

∥ ∥d (A 2u ) ∥ ∥ Eg<br />

〉<br />

〈<br />

ϕ(Eg , x) ∣ ∣d (A 2u) ∣ ∣ ϕ(E u , y) 〉 = 1 〈 E g<br />

∥ ∥ d (A 2u) ∥ ∥ E g<br />

〉<br />

〈<br />

ϕ(Eg , y) ∣ ∣d (A 2u) ∣ ∣ ϕ(E u , y) 〉 = 0<br />

〈<br />

∣<br />

ϕ(E g , i)<br />

〈<br />

∣<br />

ϕ(E g , i)<br />

∣d (Eu)<br />

x<br />

∣d (Eu)<br />

y<br />

〉<br />

∣ ϕ(E u , j)<br />

〉<br />

∣ ϕ(E u , j)<br />

= 0<br />

= 0<br />

Hier muss man also statt 12 Matrixelementen nur ein reduziertes Matrixelement<br />

berechnen.<br />

Physikalisch bedeuten die Ergebnisse, <strong>das</strong>s Zustände, die sich nach einer<br />

der Darstellungen A 1g , A 2g , B 1g , B 2g transformieren, nur dann erreicht<br />

werden können, wenn <strong>das</strong> anregende Lich in x,y-Richtung polarisiert<br />

ist. Zustände hingegen, die sich nach der irreduziblen Darstellung<br />

E g transformieren, können von E u aus nur erreicht werden, wenn <strong>das</strong><br />

Anregungslicht in z-Richtung polarisiert ist.<br />

6.1.2 Ramanübergänge<br />

Nachdem wir einige Matrixelemente für Dipolübergänge berechnet haben,<br />

wollen wir uns jetzt noch einer anderen Art von Übergängen zuwenden, den<br />

Ramanübergängen. Wie bereits in den quantenmechanischen Gr<strong>und</strong>lagen erläutert,<br />

basiert dieser Eekt auf der Polarisierbarkeit des betrachteten Moleküls.<br />

Daher muss hier, für die Berechnung der Matrixelemente, der Polarisierbarkeitstensoroperator<br />

α verwendet werden. Dies ist ein symmetrischer<br />

Tensor 2. Stufe <strong>und</strong> transformiert sich daher unter O(3) nach der symmetrisierten<br />

Produktdarstellung [ D (l=1) ⊗ D (l=1)] (g). Die Charaktere dieser<br />

+<br />

Produktdarstellung können über folgende Formel berechnet werden:<br />

χ [l⊗l] 1<br />

[<br />

]<br />

+ (g) = (χ (l) (g)) 2 + χ (l) (g 2 )<br />

2<br />

(98)<br />

Nun sollen Ramanübergänge in D 4h -Symmetrie, von einem Anfangszustand<br />

|ϕ i 〉 = |ϕ(E g , j)〉 aus, untersucht werden.<br />

Symmetriegruppe D 4h : Die Charaktertafel von D 4h haben wir bereits<br />

oben gesehen. Jetzt müssen wir noch die Charaktere der symmetrisierten<br />

35


Produktdarstellung bezüglich D 4h bestimmen. Die Charaktere der Darstellung<br />

D (l=1) (g) wurden ebenfalls bereits oben besprochen. Daraus lassen sich<br />

dann nach Gleichung (98) die Charaktere der symmetrisierten Produktdarstellung<br />

bestimmen. Man erhält:<br />

D 4h E 2 C 4 C 2 2C ′ 2 2 C ′′<br />

2 i 2 S 4 σ h 2 σ v 2 σ d<br />

D (l=1) | D4h 6 0 2 2 2 6 0 2 2 2<br />

Damit lässt sich die Zerlegung in irreduzible Darstellungen berechnen. Es<br />

ergibt sich:<br />

[<br />

D (l=1) ⊗ D (l=1)] + (g) = 2A 1g ⊕ B 1g ⊕ B 2g ⊕ E g (99)<br />

Die zugehörigen Basisfunktionen haben folgende Gestalt:<br />

A (1)<br />

1g : 1 √3 α zz ⇋ u (1) (A 1g )<br />

A (2)<br />

1g : 1<br />

2 √ 2 (α xx + α yy ) ⇋ u (2) (A 1g )<br />

B 1g :<br />

1<br />

2 (α xx − α yy ) ⇋ u(B 1g )<br />

B 2g : α xy = α yx ⇋ u(B 2g )<br />

E g :<br />

{<br />

αxz = α zx<br />

α yz = α zy<br />

⇋<br />

⇋<br />

u x (E g )<br />

u y (E g )<br />

Dabei transformieren sich die Komponenten des Polarisierbarkeitstensors α ij<br />

wie die kartesischen Produktfunktionen x i x j .<br />

Von einem Anfangszustand aus, der sich nach der Darstellung E g transformiert,<br />

sind also Endzustände erreichbar, die sich nach einer der irreduziblen<br />

Darstellungen der Produktdarstellung transformieren.<br />

(2A 1g ⊕ B 1g ⊕ B 2g ⊕ E g ) ⊗ E g = 2E g<br />

}{{}<br />

2A 1g ⊗E g<br />

⊕<br />

E g ⊕ E g ⊕<br />

}{{} }{{}<br />

B 1g ⊗E g B 2u ⊗E g<br />

A 1g ⊕ A 2g ⊕ B 1g ⊕ B 2g<br />

} {{ }<br />

E g⊗E g<br />

= 4E g ⊕ A 1g ⊕ A 2g ⊕ B 1g ⊕ B 2g(100)<br />

36


Die Endzustände können sich also nur nach folgenden irreduziblen Darstellungen<br />

transformieren (nach allen mit gerader Parität):<br />

E g , A 1g , A 2g , B 1g , B 2g<br />

Alle Matrixelemente mit anderen Endzuständen verschwinden.<br />

Beispielhaft sollen jetzt die Matrixelemente für einen Endzustand berechnet<br />

werden, der sich nach A 1g transformiert. Ein solcher Zustand kann in diesem<br />

Fall nur vom Produkt der Basisfunktionen zu E g mit anderen Basisfunktionen<br />

zu E g aufgebaut werden. Wir benötigen also wieder die Kopplungskoef-<br />

zienten für {u x (E g ), u y (E g )} ⊗ {v x (E g ), v y (E g )} in D 4h -Symmetrie. Diese<br />

sind im obigen Beispiel bereits angegeben. Sie seien an dieser Stelle nur für<br />

die hier interessante irreduzible Basisfunktion w(A 1 ) wiederholt.<br />

D 4h u x (E u )v x (E u ) u x (E u )v y (E u ) u y (E u )v x (E u ) u y (E u )v y (E u )<br />

w(A 1g ) √2 1<br />

0 0 √2 1<br />

Für den Übergang E g ↔ A 1g sind also von den zwölf möglichen Matrixelementen<br />

nur zwei verschieden von Null:<br />

〈<br />

∣<br />

ϕ(A 1g )<br />

〈<br />

∣<br />

ϕ(A 1g )<br />

∣α x<br />

(Eg)<br />

∣α y<br />

(Eg)<br />

〉 〈 ∥<br />

∣ ϕ(E g , j) = √2 1<br />

δ ∥ 〉<br />

jx A1g ∥α (E g) A1g<br />

〉 〈 ∥<br />

∣ ϕ(E g , j) = √2 1<br />

δ jy A1g α (Eg)∥ 〉<br />

∥ A 1g<br />

Da <strong>das</strong> reduzierte Matrixelement in beiden Fällen <strong>das</strong> gleiche ist, muss man<br />

also insgesamt für diesen Übergang anstatt zwölf nur ein reduziertes Matrixelement<br />

berechnen.<br />

6.2 Eigenwertprobleme<br />

Ein in vielen Bereichen der Physik sehr häug auftretendes Problem besteht<br />

im Lösen von Eigenwertproblemen. Wie wir gleich sehen werden, folgt bei<br />

Eigenwertproblemen die Blockdiagonalform einer Matrix in einer symmetrieangepassten<br />

Basis, als einfacher Spezialfall, direkt aus dem <strong>Wigner</strong>-<strong>Eckart</strong>-<br />

<strong>Theorem</strong>. Zwar kann mit Hilfe des <strong>Wigner</strong>-<strong>Eckart</strong>-<strong>Theorem</strong>s im Allgemeinen<br />

keine vollständige Diagonalisierung erreicht werden, wohl aber Blockdiagonalform.<br />

Somit wird <strong>das</strong> groÿe Eigenwertproblem in einige kleine Eigenwertprobleme<br />

zerlegt. Dies ist in den meisten Fällen, vom Rechenaufwand her,<br />

eine beträchtliche Vereinfachung.<br />

Beispielsweise bei der Lösung der Schrödingergleichung liegt ein solches Problem<br />

vor:<br />

H |Ψ〉 = E |Ψ〉 (101)<br />

37


Dabei ist H der Hamiltonoperator des Systems, E der zugehörige Energieeigenwert<br />

<strong>und</strong> |Ψ〉 eine Eigenfunktion zu E. Der Hamiltonoperator sei nun<br />

invariant unter der Gruppe G, die auf den Hilbertraum V der quadratintegrablen<br />

Funktionen wirkt. Also ist:<br />

D(g)HD(g) −1 = H (102)<br />

Damit ist H ein irreduzibler Tensoroperator 0. Ordnung <strong>und</strong> transformiert<br />

sich nach der trivialen Darstellung. Transformiert man <strong>das</strong> Eigenwertproblem<br />

in eine, der Symmetrie angepassten, Basis {|αs α i〉} <strong>und</strong> wendet von links<br />

〈βs β j| auf die Gleichung an, so erhält man:<br />

∑<br />

〈βs β j| H |αs α i〉 〈αs α i |Ψ〉 = E 〈βs β j| Ψ〉 (103)<br />

α,s α,i<br />

Das <strong>Wigner</strong>-<strong>Eckart</strong>-<strong>Theorem</strong> lautet für diesen Fall:<br />

〈βs β j| H |αs α i〉 =<br />

Vereinfacht ergibt sich:<br />

(1α|β)=δ<br />

∑ αβ<br />

s=1<br />

〈βs β ‖H‖ αs α 〉 s<br />

〈β, j|11, αi〉<br />

} {{ }<br />

=δ αβ δ ij<br />

(104)<br />

〈βs β j| H |αs α i〉 = 〈 s ′ α ‖H‖ s α<br />

〉<br />

δαβ δ ij (105)<br />

D.h. die zu diagonalisierende Matrix zerfällt in Blöcke, von denen einige<br />

sogar noch gleich sind! Man erhält also <strong>das</strong> vereinfachte Problem:<br />

∑<br />

s α<br />

〈<br />

s<br />

′<br />

α ‖H‖ s α<br />

〉<br />

〈αsα i |Ψ〉 = E 〈βs β j| Ψ〉 (106)<br />

Zur Veranschaulichung wollen wir uns nun die Matrix (〈s ′ α ‖H‖ s α 〉) nochmal<br />

in ihrer Blockdiagonalform anschauen.<br />

⎛<br />

⎜<br />

⎝<br />

〈s ′ 1 ‖H‖ s 1〉 0 · · · 0 · · · · · · 0<br />

0<br />

...<br />

.<br />

.<br />

...<br />

.<br />

0 〈s ′ i ‖H‖ s i〉 0<br />

.<br />

...<br />

.<br />

.<br />

... 0<br />

0 · · · · · · 0 · · · 0 〈s ′ r ‖H‖ s r 〉<br />

⎞<br />

⎟<br />

⎠<br />

(107)<br />

38


Die Matrix besteht jetzt aus d 1 identischen Blöcken, die zur irreduziblen<br />

Darstellung D (1) gehören, sie haben daher die Dimension m 1 . Wobei m 1<br />

die Anzahl der invarianten Unterräume des Hilbertraumes ist, die sich nach<br />

der irreduziblen Darstellung "1"transformieren. Dann aus d 2 identischen<br />

Blöcken zur irreduziblen Darstellung D (2) , usw. Man erhält also r verschiedene<br />

Blöcke, wenn r die Anzahl der verschiedenen irreduziblen Darstellungen<br />

von G ist. D.h. man muss jetzt nicht mehr eine ( ∑ r<br />

α=1 d αm α ) ×<br />

( ∑ r<br />

α=1 d αm α )-Matrix diagonalisieren, sondern nur noch r (m α ×m α )-Matrizen<br />

(wobei α = 1, . . . , r). Dies bewirkt in der Praxis eine enorme Ersparnis an<br />

Rechenaufwand <strong>und</strong> macht die Lösung mancher Probleme, bei denen Matrizen<br />

sehr hoher Dimension auftreten, überhaupt erst möglich.<br />

Nebenbemerkung: Symmetriebedingte Entartung In jedem dieser Blökke<br />

erhält man im Allgemeinen m α verschiedene Eigenwerte. Da jedoch d α identische Blöcke<br />

auftreten, sind diese Eigenwerte jeweils d α-fach entartet. Hier spricht man von symmetriebedingter<br />

Entartung, da sie alleine durch die Symmetrie des Problems gegeben ist 3 . Es<br />

kann jedoch nicht prinzipiell ausgeschlossen werden, <strong>das</strong>s beispielsweise Eigenwerte, die<br />

zu unterschiedlichen irreduziblen Darstellungen gehören, zufällig zusammenfallen. Hier<br />

spricht man von zufälliger Entartung. Eine solche tritt z.B. auf, wenn man die Beträge<br />

der Eigenwerte durch einen von Aussen vorgegebenen Parameter steuern kann, wie z.B.<br />

durch ein angelegtes Magnetfeld. Bei geeigneter Magnetfeldstärke kann es so zur Überschneidung<br />

von Eigenwerten kommen. Eine andere Möglichkeit, wie es zu einer weiteren<br />

Entartung kommen kann, ist die, <strong>das</strong>s die betrachtete Symmetriegruppe nicht die vollständige<br />

Symmetriegruppe des Systems ist.<br />

3<br />

Wir haben keine physikalischen Kenntnisse des Systems in die Betrachtung einieÿen<br />

lassen.<br />

39


7 Zusammenfassung<br />

Im Rahmen dieser Ausarbeitung haben wir gesehen, wie man die Berechnung<br />

von Matrixübergangselementen durch Symmetriebetrachtungen vereinfachen<br />

kann. Ein sehr mächtiges Werkzeug hierfür ist <strong>das</strong> <strong>Wigner</strong>-<strong>Eckart</strong>-<strong>Theorem</strong>,<br />

mit dessen Hilfe man die Berechnung von Matrixelementen in einen Teil<br />

zerlegen kann, der nur noch von der Symmetrie des Problems abhängt <strong>und</strong><br />

in einen Teil in dem die physikalische Information enthalten ist. Man kann so<br />

allein durch Betrachtung der Symmetrie schon voraussagen, <strong>das</strong>s bestimmte<br />

Matrixelemente verschwinden. Daraus lassen sich dann <strong>Auswahlregeln</strong> für<br />

bestimmte Übergänge ableiten.<br />

40


Literatur<br />

[1] M. Wagner, Gruppentheoretische Methoden in der Physik<br />

Ÿ9<br />

[2] W. Ludwig <strong>und</strong> C. Falter, Symmetries in Physics Ÿ5 - Ÿ8<br />

[3] H.-R. Trebin, Vorlesungsaufschrieb "Gruppentheoretische<br />

Methoden in der Physik"<br />

[4] C. Cohen-Tannoudji, B. Diu, F. Laloë, Quantenmechanik 1 & 2<br />

[5] M. Hamermesh, Group Theory an its Application to Physical<br />

Problems<br />

41

Hurra! Ihre Datei wurde hochgeladen und ist bereit für die Veröffentlichung.

Erfolgreich gespeichert!

Leider ist etwas schief gelaufen!